Capítulo 1

1.1

  1. \(h(\theta_1)=1/10;\ h(\theta_2)=7/10;\ h(\theta_3)=2/10.\)
  2. \(h(\theta_1|A)=0.1189;\ h(\theta_2|A)=0.7225;\ h(\theta_3|A)=0.1586.\)
  3. \(h(\theta_1|A \cap A)=0.1396;\ h(\theta_2|A \cap A)=0.7363; \ h(\theta_3|A \cap A)=0.1241.\)
  4. Não.

1.3

  1. \(P(0)=0.6321;\ P(A)=0.2326;\ P(B)=0.0855;\ P(AB)=0.0498.\)
  2. \(P(0)=P(A)=P(B)=P(AB)=0.25.\)

1.4

\(a=E(\theta)=50;\ b^2=Var(\theta)=100/6.\)

1.5

\(n \geq 66;\ n \geq 714.\)

1.6

  1. \(Ga(20,6)\).
  2. \((2.1129,4.5232)\); \((1.9431,4.8199)\).
  3. \(14.58.\)

1.8

  1. \(g(r,c|\theta)=n(n-1) r^{n-2},\ \ (r,c): \theta-\frac{1-r}{2}<c<\theta+\frac{1-r}{2},\ 0 \leq r \leq 1\) \(R|\theta \sim Be(n-1,2).\)
  1. \(g(c|\theta)= \begin{cases} n \left[1-2(\theta -c) \right]^{n-1},& \theta-1/2<c \leq \theta \\ n \left[1-2(c - \theta) \right]^{n-1},& \theta \leq c < \theta + 1/2 \end{cases}.\)
  1. O IC condicional em b) está contido no intervalo de valores positivos da verosimilhança e tem probabilidade a posteriori igual a \(\gamma\), contrariamente ao IC não condicional em c).

1.9

  1. \(\widehat{\theta}(X)=I_{\{2,3\}}(X)\), para ambos os casos.
  2. Funções críticas \(\phi(x)\) dos 4 testes MP: \(\phi_1(x)=0;\) \(\phi_2(x)= I_{\{3\}}(x);\) \(\phi_3(x)= I_{\{2,3\}}(x);\) \(\phi_4(x)=1.\)
\(E_1\): \(\qquad\qquad\qquad\) \(\phi_1\qquad\) \(\phi_2\qquad\) \(\phi_3\qquad\) \(\phi_4\quad\)
P(erro tipo I) 0 0.050 0.10 1
P(erro tipo II) 1 0.145 0.09 0
\(E_2\): \(\qquad\qquad\qquad\) \(\phi_1\qquad\) \(\phi_2\qquad\) \(\phi_3\qquad\) \(\phi_4\qquad\)
P(erro tipo I) 0 0.01 0.74 1
P(erro tipo II) 1 0.829 0.026 0
  1. \(O(H_0, H_1|x)= \begin{cases} 10,& x=1 \\ 0.91,& x=2\\ 0.06, &x=3 \end{cases}\), para ambos os casos.

1.10

  1. \((N, \sum_{i=1}^N X_i)\) é suficiente mínima com respeito ao modelo amostral para \(E_1\), \[ \{f(n,x_1,\ldots,x_n|\phi_1,\phi_2,\theta)=\phi_n \theta^{\sum_{i=1}^n x_i} (1-\theta)^{n-\sum_{i=1}^n x_i},\ \phi_n, \theta \in (0,1) \}, \] onde \(\sum_{n=1}^3 \phi_n=1\) e sendo \(N\) ancilar parcial para \(\theta\). \(Var(\sum_{i=1}^N X_i/N|\{\phi_n\}, \theta)=\theta(1-\theta)(\phi_1+\phi_2/2+\phi_3/3).\)
  2. Não, atendendo a que o modelo amostral para \(E_2\) é \[f(n,x_1,\ldots,x_n|\theta)= \begin{cases} 1-\theta,& n=1, x_1=0 \\ \theta (1-\theta),& n=2, x_1=1, x_2=0\\ \theta^2 (1-\theta),& n=3, x_1=x_2=1, x_3=0\\ \theta^3,& n=3, x_1=x_2=x_3=1.\\ \end{cases}\]
  3. Sim, independentemente de os \(\phi_n\) (supostamente distintos de \(\theta\)) serem conhecidos ou desconhecidos. Não. Sim.

1.11

  1. \(\Gamma_m(z^m)=I_{\left[1, +\infty \right)}(t_m).\)
  2. Resposta afirmativa pois \[L(\theta|n,z^n)=\prod_{j=1}^{n-1} \left[1-\Gamma_j(z^j)\right]\Gamma_n(z^n) \times \prod_{i=1}^{n} \theta^{x_i} (1-\theta)^{1-x_i} f(t_i|\theta),\] com \(T_i|\theta \sim Exp(a(\theta)),\ a(\theta)=\begin{cases} 1/5,& \theta=0.49 \\ 5,& \theta=0.51 \end{cases}\)

1.13

  1. Resposta afirmativa.

1.15

  1. Não.
  2. As inferências bayesianas sobre \(\theta\) não usam a parte da verosimilhança dependente de \(\phi\), contrariamente à generalidade das inferências frequencistas pela inaplicabilidade do PSG e do PCG.

1.16

O PCG justifica a irrelevância da distribuição marginal do vetor de frequências marginais pelo seu caráter de estatística ancilar parcial para o vetor paramétrico de interesse \(\{\theta_{(i)j}=\theta_{ij}/\theta_{i \cdot}\}\) que, em conjunto com \(\{\theta_{i \cdot}\}\), define uma transformação biunívoca de \(\theta\).

1.17

  1. \(\widetilde{\phi}=\left(\frac{2n}{\sum_{i=1}^n X_i^2}\right)^{1/2}\), violando o PSG.
  2. \(h(\phi|x_1,\ldots,x_n)\) só depende dos \(\{x_i\}\) através do valor de \(U\).

1.19

Resposta afirmativa já que \(\#{\cal X}_t= {n+t-1 \choose t}.\)

1.20

  1. Sugestão: Mostre que \(Cov (X_i, X_j)= Var \left[E(X_i|Y)\right], \forall i \neq j.\)

1.22

  1. Sugestão: Determine a distribuição do número de brancas em \(n\) extrações (distribuição de Polya) e use o método de indução.
  2. Basta considerar as variáveis \(X_1\), \(X_2\) e \(X_3\).
  3. Estenda-se o argumento de a).
  4. A distribuição de Polya é a distribuição da mistura Binomial-Beta de parâmetros \(n,\ \alpha=a/c\) e \(\beta=b/c\).

1.24

  1. Deitar fora a caixa qualquer que seja o resultado do ensaio, e não há qualquer vantagem na realização deste. A decisão minimax passa a ser vender a caixa em qualquer circunstância.
    1. A decisão Bayes é vender a caixa em qualquer circunstância, e a realização do ensaio não se justifica.

1.25

  1. \(L(\theta,a_1)= 20000\, \theta; L(\theta,a_2)= 1200 +3800\, \theta.\)
  2. Ação Bayes = \(a_2\). \(R(\theta,d_c)=\begin{cases} k (\theta-\theta_L) F_{X|\theta}(c),& \theta>\theta_L \\ k (\theta_L-\theta) \left[1-F_{X|\theta}(c)\right],& \theta \leq \theta_L \end{cases}\), onde \(k=9980 \times 1.62\).

Capítulo 2

2.4

  1. \(\alpha =\frac{1+m (\beta-2)}{1-m} \equiv \alpha(m,\beta)\). Note-se que a distribuição a priori é unimodal, sendo \(m\) o ponto de máximo se \(\alpha, \beta > 1\), o que se admitirá doravante. A equação em \(\beta\) resulta do integral definidor de \(p\) com substituição na função integranda de \(\alpha\) por \(\alpha(m,\beta)\).
  2. O valor de \(\alpha\) obtém-se de \(m\) e \(\beta\) pela formúla indicada em b). A equação em \(\beta\) pode ser resolvida por métodos numéricos. A seguinte função permite obter os valores de \(\alpha\) e \(\beta\) para quaisquer \(m,p,u\), desde que a solução exista.
fun <- function(m,p,u){
  funcao1 <- function(beta){
     alfa = (1+m*(beta-2))/(1-m)
     p-pbeta(u,alfa,beta)
  }
b <- uniroot(funcao1,c(1,40))
alfa <- (1+m*(b$root-2))/(1-m)
beta <- b$root
return(list(b,alfa,beta))
}

Aplicando aos valores dados tem-se

res <- fun(0.7,0.95,0.9)
res
## [[1]]
## [[1]]$root
## [1] 2.345704
## 
## [[1]]$f.root
## [1] 3.901604e-08
## 
## [[1]]$iter
## [1] 9
## 
## [[1]]$init.it
## [1] NA
## 
## [[1]]$estim.prec
## [1] 6.103516e-05
## 
## 
## [[2]]
## [1] 4.139976
## 
## [[3]]
## [1] 2.345704

Os valores de \(\alpha\) e \(\beta\) são ambos maiores que 1. Pode-se confirmar que na realidade a moda é \(m=0.7\) e que \(p=0.95\).

(res[[2]]-1)/(res[[2]]+res[[3]]-2)
## [1] 0.7
pbeta(0.9,res[[2]],res[[3]])
## [1] 0.95
  1. Note-se da errata que \(m=0.25\) e não \(m=0.35\). Aplicando a função definida anteriormente,
# res1 <- fun(0.25, 0.30, 0.31)

##    Error in uniroot(funcao1, c(1, 40)) :
##    f() values at end points not of opposite sign

O resultado não se altera mesmo que se aumente o limite superior do intervalo c(1,40) onde a raiz possa existir. e) \[ \begin{array}{lll} p_1 &=& \int_0^{q_1} \frac{\Gamma(\alpha+\beta)}{\Gamma(\alpha)\Gamma(\beta)}\theta^{\alpha-1}(1-\theta)^{\beta-1}d\theta \\ p_2 &=& \int_0^{q_2} \frac{\Gamma(\alpha+\beta)}{\Gamma(\alpha)\Gamma(\beta)}\theta^{\alpha-1}(1-\theta)^{\beta-1}d\theta \end{array} \] Pretende-se obter \(\alpha\) e \(\beta\) para \(p_1=0.25, q_1=0.7, p_2=0.75, q_2=0.9\) O seguinte código permite obter a solução usando a função BBsolve() do pacote BB.

fun1 <- function(p1,q1,p2,q2,z){
  func2 <- function(x){
    f<-c(p1-pbeta(q1,x[1],x[2]),p2-pbeta(q2,x[1],x[2]))
}
library(BB)
sol <- BBsolve(z, func2)
sol
}

No argumento de fun1, além de \(p_1, q_1, p_2, q_2\), aparece um vetor \(z\) de dimensão 2 que deve conter valores iniciais para a solução do sistema. Aplicando ao exemplo e considerando, por exemplo, \(z=c(2.5,3.5)\) obtém-se

sol <- fun1(0.25,0.7,0.75,0.9,c(2.5,3.5))
## Warning in pbeta(q1, x[1], x[2]): NaNs produced
## Warning in pbeta(q2, x[1], x[2]): NaNs produced
##   Successful convergence.
sol
## $par
## [1] 5.537182 1.503637
## 
## $residual
## [1] 2.741368e-08
## 
## $fn.reduction
## [1] 2.479161e-05
## 
## $feval
## [1] 97
## 
## $iter
## [1] 16
## 
## $convergence
## [1] 0
## 
## $message
## [1] "Successful convergence"
## 
## $cpar
## method      M     NM 
##      2     50      1

Os valores de \(\alpha\) e \(\beta\) encontram-se em

sol$par[1]  # alpha
## [1] 5.537182
sol$par[2]  # beta
## [1] 1.503637

Usando o pacote SHELF

library(SHELF)
vec <- c(0.7,0.9)
p1 <- c(0.25,0.75)
fit.good <- fitdist(vals = vec, probs = p1, lower = 0, upper = 1)
fit.good$Beta
##     shape1   shape2
## 1 5.537321 1.503659

2.5

  1. Sendo \(\theta\sim D_k(\alpha)\), a densidade marginal de \(\theta_i\) para todo o \(i\) é \[ h(\theta_i|\alpha)=\int_{\theta_{(-i)}\in (0,1)^{k-1}}\frac{\Gamma(\sum_{j=1}^{k+1}\alpha_j)}{\prod_{j=1}^{k+1}\Gamma(\alpha_j)} \prod_{j=1}^{k}\theta_j^{\alpha_j-1} (1-\sum_{j=1}^k \theta_j)^{\alpha_{k+1}-1}d\theta_{(-i)} \] onde \(\theta_{(-i)}=(\theta_j,j=1,...,k,j\neq i)\), isto é, é o vetor \(\theta\) excluindo a \(i\)-ésima componente e \((0,1)^{k-1}\) é o produto cartesiano do intervalo \((0,1)\), \(k-1\) vezes. Para calcular aquele integral múltiplo faça-se a seguinte transformação de variáveis \(\theta_j=u_j(1-\theta_i), \quad \forall j\neq i\), obtendo-se \[ h(\theta_i|\alpha)=\frac{\Gamma(\sum_{j=1}^{k+1}\alpha_j)}{\prod_{j=1}^{k+1}\Gamma(\alpha_j)}\theta_i^{\alpha_i-1} (1-\theta_i)^{\sum_{j\neq i}\alpha_j-1}\int_{u_{(-i)}\in (0,1)^{k-1}} \prod_{j\neq i}u_j^{\alpha_j-1} (1-\sum_{j\neq i} u_j)^{\alpha_{k+1}-1}du_{(-i)} \] onde \(u_{(-i)}\) é o vetor dos \(u_j,j\neq i\). O último integral é o núcleo de uma densidade Dirichlet para o vetor \(u_{(-i)}\), sendo portanto igual a \[ \frac{\prod_{j\neq i}\Gamma(\alpha_j)}{\Gamma(\sum_{j\neq i}\alpha_j)}, \] e consequentemente \[ h(\theta_i|\alpha)=\frac{\Gamma(\sum_{j=1}^{k+1}\alpha_j)}{\Gamma(\alpha_i)\Gamma(\sum_{j\neq i}\alpha_j)}\theta_i^{\alpha_i-1}(1-\theta_i)^{\sum_{j\neq i}\alpha_j-1}, \] evidenciando que \(\theta_i\sim Be(\alpha_i,\alpha_.-\alpha_i)\) onde \(\alpha_.=\sum_{i=1}^{k+1} \alpha_i.\)
  2. Com base na informação dada pode admitir-se o seguinte, relativamente aos quantis de probabilidade \(p_1=0.25\) e \(p_2=0.75\) da distribuição a priori Beta para cada um dos parâmetros (\(\theta_1,\theta_2,\theta_3\)) das distribuições Binomiais \(Bi(10, \theta_i)\),
\(\qquad\qquad\qquad\) \(\theta_1\quad\quad\) \(\theta_2\quad\quad\) \(\theta_3\quad\quad\)
\(p_1=0.25\) 0.4 0.2 0.1
\(p_2=0.75\) 0.6 0.4 0.3

Para encontrar os parâmetros (\(\alpha_i,\beta_i\), i=1,2,3) das distribuições Beta pode usar-se a função referida no problema anterior ou a função do pacote SHELF. Obtém-se então

\(\qquad\qquad\) \(\theta_1\qquad\qquad\) \(\theta_2\qquad\qquad\) \(\theta_3\qquad\qquad\)
\(\alpha_i\) 5.815778 2.963769 1.503659
\(\beta_i\) 5.81578 6.691304 5.537321
  1. Como o valor médio de uma \(Be(\alpha, \beta)\) é \(\frac{\alpha}{\alpha+\beta}\), obtém-se então \[ E_1=0.4999999, E_2=0.3069649, E_3=0.2135581, \] implicando \(d=E_1+E_2+E_3= 1.020523\). Como \(d\neq 1\) estas médias são incompatíveis com a distribuição Dirichlet para \(\theta\), pelo que terá de haver ajuste na eliciação. Considerem-se novos valores para \(\alpha_i\) e \(\beta_i\) usando as equações \[ \alpha_i^* = \frac{\alpha_i}{d} \quad \text{e} \quad \beta_i^* = \alpha_i+\beta_i-\alpha_i^*, \] que implicam para as médias correspondentes as relações \(\mu_i^*=E_i/d\). Para que os \(\mu_i^*\) possam traduzir médias a priori dos \(\theta_i\), ou seja, \(\mu_i^*=\alpha_i^*/\sum_j \alpha_j^*\), é necessário que \(\alpha_i^*+\beta_i^*\) não dependa de \(i\). Ora
\(\alpha_1^*\qquad\qquad\) \(\beta_1^*\qquad\qquad\) \(\alpha_2^*\qquad\qquad\) \(\beta_2^*\qquad\qquad\) \(\alpha_3^*\qquad\qquad\) \(\beta_3^*\qquad\qquad\)
5.698821 4.377586 2.904167 7.172241 1.47342 8.602988
de modo que \[\begin{eqnarray*} \alpha_1^*+\beta_1^* &=& 11.63156 \\ \alpha_2^*+\beta_2^* &=& 9.655073 \\ \alpha_3^*+\beta_3^* &=& 7.04098 \end{eqnarray*}\]

Neste ponto deve-se procurar um valor de compromisso. Tome-se, por exemplo, a média \(m=9.442538\) desses três valores e considere-se \[ \alpha_i^{**}= m \mu_i^* \equiv m \frac{\alpha_i^*}{\alpha_1^*+\beta_i^*}, \quad i=1,2,3. \] Então \[ \beta_i^{**}=m-\alpha_i{**}, \quad i=1,2,3, \] tendo-se portanto os parâmetros das distribuições marginais discriminados abaixo compatíveis com uma distribuição conjunta Dirichlet,

\(\alpha_1^{**}\) \(\beta_1^{**}\) \(\alpha_2^{**}\) \(\beta_2^{**}\) \(\alpha_3^{**}\) \(\beta_3^{**}\)
4.626322 4.816215 2.840238 6.602299 1.975978 7.466559

O pacote SHELF adapta este procedimento usando, não a média, mas uma escolha otimizada de \(m\). Sugere-se que implemente esse procedimento, o qual se encontra descrito em Zapata-Vázquez et al. (2014).

2.6

  1. Sim; 2.

2.7

  1. Tome-se \(\psi(\theta)= e^{\sum_{j=k+1}^{k+s}Q_j(\theta)a_j}\), para \((a_{k+1}, \ldots,a_{k+s})\) apropriado.
  2. \(h(\theta|x)=\sum_{i=1}^{m} w_i^* h_i(\theta|x),\ w_i^*= \frac{w_i f_i(x)}{\sum_{j=1}^{m}w_j f_j(x)},\ f_i(x)=\frac{h_i(\theta) f(x|\theta)}{h_i(\theta|x)}.\)

2.9

  1. \(I(\theta)=c \Rightarrow\) verificação de a).
  2. \(I(\theta)\propto \theta^{-2} \Rightarrow\) verificação de b).

2.10

  1. \(\psi=\sqrt{\theta}\).
  2. \(\psi=\ln \frac{1-\sqrt{1-\theta}}{1+\sqrt{1-\theta}}\).
  3. \(\psi=\ln{\theta}.\)

2.11

\(h(\theta) \propto [I(\theta)]^{1/2} = \frac{1}{\theta}\).

2.12

  1. \(h(\theta) \propto \prod_{i=1}^{c}\theta_i^{-1/2}\)
    (Sugestão: \((D_{\theta}-\theta \theta')^{-1}= D_{\theta}^{-1}+\frac{1_{c-1} 1_{c-1}'}{1-1_{c-1}'\theta}\); \(|D_{\theta}-\theta \theta'|=\prod_{i=1}^{c}\theta_i,\ \theta_c=1-1_{c-1}'\theta).\)

2.13

  1. \(h(\theta) \propto 1/\sqrt{\theta},\,\theta>0\).
  2. \(\psi \propto \sqrt{\theta}.\)

2.15

  1. \(\theta|N,x \sim Be(x+1, N-x+1); \ h(N|x)\propto (N+1)^{-1}, N \geq x\) \(N|\theta,x \stackrel{\rm d}{\equiv} M-1|\theta,x\), com \(M|\theta,x \sim BiN(x+1, \theta)\).
  2. \(h(\theta|x)\propto \theta^{-1} I_{(0,1)}(\theta).\)

2.16

  1. \(f(z|\epsilon,\delta)= \frac{\displaystyle{c^{n-\epsilon}\Gamma(n)}}{\displaystyle{(1+\sum_{i=2}^{\epsilon}z_i+c\sum_{i=\epsilon+1}^{n}z_i)^n}} \equiv f(z|\epsilon),\ z_i>0,\,i=2,\ldots,n.\)
    A ancilaridade específica de \(Z\) para \(\delta\) era previsível porque o modelo amostral faz parte da família de escala.

  2. \(h(\epsilon|w,z) \propto h(\epsilon) f(z|\epsilon) (1+\sum_{i=2}^{\epsilon}z_i+c\sum_{i=\epsilon+1}^{n}z_i)^{k-1}.\)

  3. Afirmação falsa em geral, como o comprova a forma de \(h(\epsilon|w,z)\). Dentro da família considerada em b) para \(h(\epsilon, \delta)\), apenas no caso \(k=1\) é tal afirmação verdadeira.

  4. Para \(h(\epsilon, \delta)\) própria e bem comportada, a condição de \(h(\epsilon|w,z) \equiv h(\epsilon;z)\) já implica que \(h(\epsilon;z) \propto h(\epsilon) f(z|\epsilon)\).
    (Sugestão: Parta-se de \(h(\epsilon|w,z) p(w,z)= \int {f(w|z,\epsilon,\delta)f(z|\epsilon) h(\epsilon, \delta)} d\delta\) e integre-se ambos os membros em ordem a \(w\)).

2.19

O objetivo é obter a função \(y=h(\theta)\) que maximize a entropia definida em (2.24) sujeita às restrições \[ \int_0^1 h(\theta) d\theta= 1,\ \ \int_0^1 \theta h(\theta) d\theta = \mu. \] Recorrendo ao cálculo variacional e ao método dos multiplicadores de Lagrange, tal como aplicados em alguns exemplos deste capítulo, essa função \(y=h(\theta)\) é solução de \(\frac{\partial F}{\partial y}=0\), com \[ F(y,\theta)=-y\ln y+y\ln h_0(\theta) +\lambda_0 y+\lambda_1 \theta y, \] obtendo-se \[ y=h_0(\theta)\exp\{-1+\lambda_0+\lambda_1 \theta\}. \] Como \(-1+\lambda_0\) não depende de \(\theta\) tem-se o resultado \[ h(\theta)\propto \theta^{-\frac{1}{2}}(1-\theta)^{-\frac{1}{2}}e^{\lambda_1\theta}\quad \theta\in(0,1). \] [Note-se que esta expressão surge como consequência imediata de (2.26) com \(j=1\) e \(g_j(\theta)=\theta\)}].

Dado que \(\int_0^1 \theta h(\theta)d\theta=\mu\) tem-se \[ \frac{\int_0^1 \theta^{1-\frac{1}{2}}(1-\theta)^{-\frac{1}{2}}e^{\lambda_1\theta}d\theta}{\int_0^1 \theta^{-\frac{1}{2}}(1-\theta)^{-\frac{1}{2}}e^{\lambda_1\theta}d\theta}=\mu. \] Usando a representação integral da função hipergeométrica generalizada \[ \int_0^u x^{\nu-1}(u-x)^{\eta-1}e^{\beta x}dx=B(\eta,\nu)_1F_1(\nu;\eta+\nu;\beta u), \] o quociente anterior pode escrever-se na forma \[ \frac{B(3/2,1/2)\, _1F_1(3/2,2,\lambda_1)}{B(1/2,1/2)\, _1F_1(1/2,1,\lambda_1)}=\mu. \] Atendendo a que \(B(a,b)=\frac{\Gamma(a)\Gamma(b)}{\Gamma(a+b)}\) e que \(\Gamma(x+1)=x\Gamma(x),\,x>0\), tem-se que \(\lambda_1\) é solução de \[ _1F_1\left(\frac{3}{2};2;\lambda_1\right)=2\mu _1F_1\left(\frac{1}{2};1;\lambda_1\right), \] como se pretendia provar. Em particular, a) para \(\mu=\frac{1}{2}\), tem-se \(\lambda_1=0\) implicando a distribuição de Jeffreys; b) para \(\mu=0.7\), \(\lambda_1\approx 1.748159\); c) para \(\mu=0.3\), \(\lambda_1\approx-1.748131\). Para obter os valores de \(\lambda_1\) correspondentes avalores particulares de \(\mu\) usou-se o seguinte código de :

f11_func<-function(a,b,c,u){
  d<-b-a
  A<-beta(d,a)
  fun_x<-function(x) x^(a-1)*(u-x)^(d-1)*exp(c*x)
  B<-integrate(fun_x,0,1,subdivisions = 300L,
            rel.tol = .Machine$double.eps^0.8)$value
  return(B/A)
  }

# the result is 1F1(a,b,c*u)

exer_fun<-function(c,mu){
  f11_func(3/2,2,c,1)-2*mu*f11_func(1/2,1,c,1)
  }

uniroot(function(c) exer_fun(c,1/2),c(-4,4))$root
## [1] 6.823789e-08
uniroot(function(c) exer_fun(c,0.7),c(-4,4))$root
## [1] 1.748159
uniroot(function(c) exer_fun(c,0.3),c(-4,4))$root
## [1] -1.748131

Estes valores foram confirmados por dois processos, nomeadamente por simulação de Monte Carlo (veja-se capítulo 7) e usando a função do pacote de , como segue:

library(fAsianOptions)
## Loading required package: timeDate
## Loading required package: timeSeries
## Loading required package: fBasics
## Loading required package: fOptions
kummerM(0,3/2,2)-2*1/2*kummerM(0,1/2,1)
## [1] 0+0i
kummerM(1.748159,3/2,2)-2*0.7*kummerM(1.748159,1/2,1)
## [1] -4.310835e-07+0i
kummerM(-1.748131,3/2,2)-2*0.3*kummerM(-1.748131,1/2,1)
## [1] 2.75751e-06+0i
# usando método de Monte Carlo simples

x<-rbeta(50000,3/2,1/2)
x1<-rbeta(50000,1/2,1/2)
num<-mean(exp(1.748159*x))
den<-2*mean(exp(1.748159*x1))
num/den   # deve dar aproximadamente 0.7
## [1] 0.7028106
num1<-mean(exp(-1.748131*x))
den1<-2*mean(exp(-1.748131*x1))
num1/den1 # deve dar aproximadamente 0.3
## [1] 0.2986602

2.20

De acordo com o método de Berger-Bernardo, uma distribuição a priori \(h(\theta)\) de referência deve maximizar a quantidade de informação remanescente definida em (2.29), ou seja \[ I^\theta\{e(\infty),h(\theta)\}=\lim_{n\rightarrow \infty}I^\theta\{e(n),h(\theta)\}. \] No caso em que \(\theta\) tem suporte finito tem-se, usando $h(_i|x)= $ \[ \begin{array}{lll} I^\theta\{e(n),h(\theta)\} &=& \int p(x)\sum_{i=1}^k h(\theta_i|x)\ln\frac{h(\theta_i|x)}{h(\theta_i)}dx \\ &=& \sum_{i=1}^k \left(\int f(x|\theta_i)h(\theta_i)\ln\frac{h(\theta_i|x)}{h(\theta_i)}dx \right)\\ &=& -\sum_{i=1}^k h(\theta_i)\ln h(\theta_i)+\sum_{i=1}^k h(\theta_i)\int f(x|\theta_i) \ln h(\theta_i|x) dx. \end{array} \] Quando \(\Theta\) é finito, para qualquer distribuição a priori estritamente positiva, a distribuição a posteriori em \(\theta_v\), \(h(\theta_v|x)\) converge para 1 quando \(n\to \infty\) e \(\theta_v\) é o verdadeiro valor do parâmetro (veja-se demonstração em Bernardo e Smith, 1994, p. 286 e Exercício 2.25). Assim o segundo termo do segundo membro da expressão anterior tende para zero quando \(n\rightarrow\infty\) e, consequentemente, a quantidade de informação remanescente reduz-se a \[ I^\theta\{e(\infty),h(\theta)\}=-\sum_{i=1}^k h(\theta_i)\ln h(\theta_i), \] ou seja, a entropia de \(h(\theta), \theta \in \Theta\), a qual é máxima para \(h(\theta_i)=\frac{1}{k}\), para \(i=1,...,k\).

2.21

Com a reparametrização considerada tem-se que \[ \gamma = \theta+\psi(1-\theta), \ \phi = \frac{\theta}{\theta+\psi(1-\theta)} \] O modelo binomial bivariado com tal reparametrização fica expresso por \[ f(r,s|\theta,\psi)={m\choose r}{r\choose s} \theta^s(1-\theta)^{m-s}\psi^{r-s}(1-\psi)^{m-r},\quad r=0,...,m \quad s=0,...,r, \] com \(\theta\in(0,1)\) e \(\psi\in(0,1)\).

  • Fixe-se \(\theta\) como parâmetro de interesse. A distribuição a priori de \(\psi\) condicional a \(\theta\) é \[ h(\psi|\theta)\propto \left[I(\psi|\theta)\right]^{1/2}, \] onde \[ \begin{array}{lll} I(\psi|\theta) &=& -E_{(R,S|\theta,\psi)}\left[ \frac{\partial^2}{\partial \psi^2}\ln f(r,s|\theta,\psi)\right]\\ &=& E_{(R,S|\theta,\psi)}\left[\frac{m-R}{(1-\psi)^2}+\frac{R-S}{\psi^2}\right] = \frac{m(1-\theta)}{\psi(1-\psi)}, \end{array} \] dado que \(E_{(R,S|\theta,\psi)}=m(\theta+\psi(1-\theta))\) e \(E_{(S|\theta,\psi)}=m\theta\). Assim \[ h(\psi|\theta)\propto \psi^{-1/2}(1-\psi)^{-1/2},\quad \psi\in(0,1), \] ou seja \(\psi|\theta \sim Beta(1/2,1/2)\). De acordo com o método descrito nas páginas 122-123, \(h(\theta)\propto \left[I(\theta)\right]^{1/2}\), com \[ \left[I(\theta)\right]=-E_{(R,S|\theta,\psi)}\left[ \frac{\partial^2}{\partial \theta^2}\ln f(r,s|\theta)\right], \] onde \[ \begin{array}{lll} f(r,s|\theta) &=& \int_0^1 f(r,s|\theta,\psi)h(\psi|\theta)d\psi\\ &=& \int_0^1 {m\choose r}{r\choose s} \theta^s(1-\theta)^{m-s}\psi^{r-s}(1-\psi)^{m-r} \frac{\psi^{-1/2}(1-\psi)^{-1/2}}{B(1/2,1/2)}d\psi \\ &\propto& \theta^s(1-\theta)^{m-s}. \end{array} \] Portanto, \[ \begin{array}{lll} I(\theta) &=& -E_{(R,S|\theta}\left[ \frac{\partial^2}{\partial \theta^2}\ln f(r,s|\theta)\right]\\ &=& E_{(R,S|\theta}\left[\frac{S}{\theta^2}+\frac{m-S}{(1-\theta)^2}\right] = \frac{m}{\theta(1-\theta)}, \end{array} \] implicando \(h(\theta)\propto \theta^{-1/2}(1-\theta)^{-1/2}\quad \theta\in(0,1)\) e \[ h(\theta,\psi)\propto \theta^{-1/2}(1-\theta)^{-1/2}\psi^{-1/2}(1-\psi)^{-1/2},\quad (\theta,\psi)\in (0,1)\times(0,1). \]

  • Tome-se agora \(\psi\) como parâmetro de interesse. A distribuição a priori de \(\theta\) condicional a \(\psi\) é \[ h(\theta|\psi)\propto \left[I(\theta|\psi)\right]^{1/2}. \] Raciocínio idêntico ao caso anterior leva a \(\theta|\psi \sim Beta(1/2,1/2).\) Por outro lado, \(h(\psi)\propto [I(\psi)]^{-1/2}\) com \[ \begin{array} I(\psi) &=& -E_{(R,S|\psi)}\left[ \frac{\partial^2}{\partial \psi^2}\ln f(r,s|\psi)\right]\\ &=& E_{(R,S|\psi)}\left[\frac{R-S}{\psi^2}+\frac{m-R}{(1-\psi)^2}\right] = \frac{m/2}{\psi(1-\psi)}, \end{array} \] dado que \[ \begin{array}{lll} f(r,s|\psi) &=& \int_0^1 f(r,s|\theta,\psi)h(\theta|\psi)d\theta \propto \psi^{r-s}(1-\psi)^{m-r} \\ E(R|\psi) &=& \int_0^1 (m\theta+m\psi(1-\theta))\frac{\theta^{-1/2}(1-\theta)^{-1/2}}{B(1/2,1/2)}d\theta=\frac{m(1+\psi)}{2} \\ E(S|\psi) &=& \int_0^1 m\theta\frac{\theta^{-1/2}(1-\theta)^{-1/2}}{B(1/2,1/2)}d\theta=\frac{m}{2}. \end{array} \] Consequentemente, \(h(\psi)\propto \psi^{-1/2}(1-\psi)^{-1/2}\) e novamente se tem \[ h(\theta,\psi)\propto \theta^{-1/2}(1-\theta)^{-1/2}\psi^{-1/2}(1-\psi)^{-1/2},\quad (\theta,\psi)\in (0,1)\times(0,1). \]

2.22

De acordo com a reparametrização em termos de (\(\psi,\phi\)) com \(\psi=\gamma\phi\), o modelo Binomial-Poisson considerado no Exemplo 2.27 é \[ f(x,y|\psi,\phi)={y\choose x} \psi^x(\phi-\psi)^{y-x}\frac{\mbox{e}^{-\phi}}{y!}, \quad y=0,1,...;x=0,...,y \] com \(\psi\in(0,\phi)\) e \(\phi>0\), donde \[ \ln f(x,y|\psi,\phi)\propto x\ln(\psi)+(y-x)\ln(\phi-\psi)-\phi. \] Sendo \(\phi\) o parâmetro perturbador, fixe-se \(\psi\). Pela Proposição 2.2, tem-se \[ h(\phi|\psi)\propto \left[I(\phi|\psi)\right]^{1/2} \] com \[ I(\phi|\psi)=-E_{X,Y|\psi,\phi}\left[\frac{Y-X}{(\phi-\psi)^2}\right]=\frac{1}{\phi-\psi}, \] implicando a distribuição imprópria \[ h(\phi|\psi)\propto(\phi-\psi)^{-1/2},\quad 0<\psi<\phi. \] Considere-se então \(\left\{\Phi_i(\psi)\right\},0<\psi<\phi\) uma sucessão crescente de subconjuntos de \(\Phi=(0,+\infty)\) de intervalos da forma \(\Phi_i(\psi)=(\psi, a_i+\psi)\) com \(a_i\) uma sucessão crescente de números reais positivos. Então \[ h_i(\phi|\psi)=\frac{(\phi-\psi)^{-1/2}}{2a_i^{1/2}}, \quad \phi \in (\psi,\psi+a_i) \] \[ f_i(x,y|\psi)={y\choose x}\psi^x \frac{\mbox{e}^{-\psi}}{2a_i^{1/2}}\Gamma(y-x+\/2,a_i). \] \[ E_i(X)=\int_\psi^{a_i+\psi}E(X|\psi,\phi)h_i(\phi|\psi)d\phi=\psi. \] Portanto \[ h_i(\psi)\propto I_i(\psi)^{1/2}=\psi^{-1/2}. \] A distribuição a priori de referência quando \(\psi\) é o parâmetro de interesse é pois \[ h(\psi,\phi)\propto \psi^{-1/2}(\phi-\psi)^{-1/2},\quad 0<\psi<\phi. \]

2.23

Aplique-se aqui a Proposição 2.3, dado que se verificam as condições nela enunciadas (veja-se subsecção 5.2.1 deste livro). O espaço paramétrico é \(\Gamma\times\Phi=\mathbb{R}\times \mathbb{R}^+\) com \(\mu\in \Gamma\) and \(\sigma\in \Phi\).

  1. \(\sigma\) é parâmetro perturbador e \(\mu\) parâmetro de interesse.
    Fixe-se \(\mu\). Como a matriz de informação de Fisher é \[ I(\mu,\sigma)=\left( \begin{array}{cc} \sigma^{-2} & 0 \\ 0 & 2\sigma^{-2} \\ \end{array} \right), \] tem-se \(h(\sigma|\mu)\propto \sigma^{-1}\), \(\sigma>0\), a qual é imprópria.
    Considere-se a sucessão \(\Phi_i(\mu)=\{\sigma: \mbox{e}^{-i}\leq\sigma\leq \mbox{e}^i\}\), \(i=1,2,...\) crescente de subconjuntos de \(\Phi\). Para qualquer \(i\) tem-se que \(h_i(\sigma|\mu)=\frac{1}{2i\sigma}, \quad \mbox{e}^{-i}\leq\sigma\leq \mbox{e}^i\), a qual é própria.
    Por outro lado, \[ \left[h_\mu(\mu,\sigma)\right]^{1/2}=\left[h_{11}-h_{12}h_{22}^{-1}h_{21}\right]^{1/2}=\sigma^{-1} \] em que \(h_{ij}\) são os elementos da matriz \(I(\mu,\sigma)\) e \(h_{22}^{1/2}=\sqrt{2}\sigma^{-1}.\) Verificando-se a fatorização indicada na página 126, com \(f_1(\mu)=1\) e \(g_2(\sigma)=\sigma^{-1}\) e a sucessão \(\Phi_i(\mu)\) não depender de \(\mu\), a distribuição a priori de referência para \((\mu,\sigma)\) é \[ h(\mu,\sigma)\propto \sigma^{-1}\quad \mu\in \mathbb{R}, \sigma>0. \]
  2. \(\mu\) é parâmetro perturbador e \(\sigma\) parâmetro de interesse.
    A matriz de informação de Fisher é agora \[ I(\sigma,\mu)=\left( \begin{array}{cc} 2\sigma^{-2} & 0 \\ 0 & \sigma^{-2} \\ \end{array} \right), \] e a distribuição a priori de referência condicional \(h(\mu|\sigma)\propto \sigma^{-1}, \mu\in \mathbb{R}\).
    Tomando a sucessão \(\Gamma_i(\sigma)=\{\mu: -\mbox{e}^{-i}\leq\mu\leq \mbox{e}^i\}\), \(i=1,2,...\), a qual não depende de \(\sigma\) e considerando a fatorização \[ \left[h_{\sigma}\right]^{1/2}=\sqrt{2}\sigma^{-1}, \quad h_{22}=\sigma^{-1}, \] tem-se novamente que a distribuição a priori de referência para \((\mu,\sigma)\) é \[ h(\mu,\sigma)\propto \sigma^{-1},\quad \mu\in \mathbb{R}, \sigma>0. \] Obtenha-se agora a distribuição a priori de Jeffreys nas diferentes situações indicadas.
  • \(\mu\) e \(\sigma\) não são independentes a priori,
    Considere-se novamente a matriz de informação de Fisher \[ I(\mu,\sigma)=\left( \begin{array}{cc} \sigma^{-2} & 0 \\ 0 & 2\sigma^{-2} \\ \end{array} \right), \] cujo determinante é \(|I(\mu,\sigma)|=2\sigma^{-4}.\)
    De acordo com a regra de Jeffreys multiparamétrica a distribuição a priori para \((\mu,\sigma)\) é então \[ h(\mu,\sigma)\propto \sigma^{-2}, \quad \mu\in \mathbb{R}, \sigma>0 \]
  • \(\mu\) e \(\sigma\) são independentes a priori
    Admitindo que \(h(\mu,\sigma)=h(\mu)h(\sigma)\) e aplicando a regra de Jeffreys uniparamétrica tem-se \[ h(\mu)\propto 1,\, \mu\in \mathbb{R}, \quad h(\sigma)\propto \sigma^{-1},\sigma>0, \] implicando \[ h(\mu,\sigma)\propto \sigma^{-1}, \quad \mu\in \mathbb{R}, \sigma>0\] assim idêntica à distribuição a priori de referência atrás obtida.

2.24

A matriz de informação de Fisher é

\[ I(\theta_1,\theta_2)=\left( \begin{array}{cc} n \frac{1-\theta_2}{\theta_1(1-\theta_1-\theta_2)} & n\frac{1}{1-\theta_1-\theta_2} \\ n\frac{1}{1-\theta_1-\theta_2} & n \frac{1-\theta_1}{\theta_2(1-\theta_1-\theta_2)} \\ \end{array} \right). \] Aplicando novamente a Proposição 2.3 tem-se (sendo \(\theta_1\) parâmetro de interesse e \(\theta_2\) parâmetro perturbador) \[ h(\theta_2|\theta_1)\propto (1-\theta_1)^{1/2}\theta_2^{-1/2}(1-\theta_1-\theta_2)^{-1/2}, \quad \theta_2<1-\theta_1. \] Esta distribuição a priori é própria pois \[ \int_0^{1-\theta_1} \theta_2^{-1/2}(1-\theta_1-\theta_2)^{-1/2}d\theta_2=B(1/2,1/2). \] Consequentemente \[ h(\theta_2|\theta_1)=\frac{\theta_2^{-1/2}(1-\theta_1-\theta_2)^{-1/2}}{B(1/2,1/2)}, \quad \theta_2<1-\theta_1. \] De acordo com a Proposição 2.3 e dado qúe \(h_{11}- h_{12}h_{22}^{-1}h_{21}=[\theta_1(1-\theta_1)]^{-1}\) não depende de \(\theta_2\), é claro que \[ \begin{array}{lll} h(\theta_1) &\propto& \exp\left\{\int_0^{1-\theta_1} h(\theta_2|\theta_1) \ln [\theta_1(1-\theta_1)]^{-1/2}d\theta_2\right\}\\ &=& [\theta_1(1-\theta_1)]^{-1/2}, \ \theta_1\in (0,1). \end{array} \] Consequentemente a distribuição a priori de referência para \((\theta_1,\theta_2)\), quando \(\theta_1\) é o parâmetro de interesse, é \[ h(\theta_1,\theta_2)=h(\theta_2|\theta_1)h(\theta_1)\propto \theta_1^{-1/2}(1-\theta_1)^{-1/2}\theta_2^{-1/2}(1-\theta_1-\theta_2)^{-1/2}, \quad \theta_1+\theta_2 < 1. \] Dado que o determinante da matriz de informação de Fisher é \[ |I(\theta_1,\theta_2)|=n(\theta_1^{-1/2}\theta_2^{-1/2}(1-\theta_1-\theta_2)^{-1/2}), \] segue-se que a distribuição a priori de Jeffreys é \[ h(\theta_1,\theta_2)\propto \theta_1^{-1/2}\theta_2^{-1/2}(1-\theta_1-\theta_2)^{-1/2}, \quad \theta_1+\theta_2 < 1. \]

2.25

  1. Tem-se que \(f(x|\theta)=\prod_{j=1}^n f(x_j|\theta)\) e seja \(\theta_v\) o verdadeiro valor do parâmetro e \(h(\theta_k)=p_k>0\), com \(\sum_{k}{p_k}=1\) uma distribuição a priori. Seja \(X=(X_1,...,X_n)\) o vetor aleatório do qual \(x=(x_1,...,x_n)\) é uma realização. Então $k $ tem-se \[ \begin{array}{lll} \frac{f(X|\theta_k)}{f(X|\theta_v)}&=&\prod_{j=1}^n \frac{f(X_j|\theta_k)}{f(X_j|\theta_v)}\\ &=& \exp\left\{\ln \prod_{j=1}^n\frac{f(X_j|\theta_k)}{f(X_j|\theta_v)}\right\} = \exp\left\{\sum_{j=1}^n \ln \frac{f(X_j|\theta_k)}{f(X_j|\theta_v)}\right\}\\ &=&\exp(\sum_{j=1}^n Y_j)=\exp(S_k). \end{array} \] Condicionalmente a \(\theta_v\), e para \(k\) fixo, \(S_k\) é a soma de \(n\) variáveis aleatórias \(Y_j\) i.i.d. com valor médio finito \(\mu\) dado por \[ \mu=E(Y)=\int \ln \frac{f(y|\theta_k)}{f(y|\theta_v)}f(y|\theta_v)dy. \] Este valor médio é negativo quando \(k\neq v\) e 0 quando \(k=v\). Pela Lei Forte dos Grandes Números, então \[ \lim_{n\to\infty}\frac{1}{n}S_k=\mu, \] o que implica que \(\lim_{n\to\infty}S_v=0\) e \(\lim_{n\to\infty} S_k=-\infty,\) para \(\forall k\neq v\). Por outro lado, $k $ \[ \begin{array}{lll} h(\theta_k|X) &=& \frac{p_kf(X|\theta_k)}{\sum_{i} f(X|\theta_i)p_i} = \frac{p_k f(X|\theta_k)/f(X|\theta_v)}{\sum_{i} f(X|\theta_i)/f(X|\theta_v)p_i} \\ &=& \frac{\exp[\ln p_k+S_k]}{\sum_i\exp[\ln p_i+S_i]}. \end{array} \] Consequentemente, quando \(n\to\infty\), \(h(\theta_k|x)\to 0\) e \(h(\theta_v|x)\to 1\).

  2. Reveja-se o Exercício 2.20.

2.26

Admita-se que a sucessão \(\{\Phi_i(\gamma)\}\) de valores de \(\phi\) não depende de \(\gamma\) e que é válida a fatorização \[ [h_\gamma(\gamma,\phi)]^{1/2}=f_1(\gamma)g_1(\phi),\quad [h_{22}(\gamma,\phi)]^{1/2}=f_2(\gamma)g_2(\phi). \] De acordo com a Proposição 2.3, \[ h(\phi|\gamma)\propto [h_{22}(\gamma,\phi)]^{1/2}= f_2(\gamma)g_2(\phi). \] Para cada \(\Phi_i(\gamma)\) tem-se a sucessão de distribuições a priori condicionais \[ \begin{array}{lll} h_i(\phi|\gamma) &=& \frac{f_2(\gamma)g_2(\phi)}{\int_{\Phi_i(\gamma)}f_2(\gamma)g_2(\phi)d\phi} \\ &=& \frac{g_2(\phi)}{\int_{\Phi_i(\gamma)}g_2(\phi)d\phi}= \frac{g_2(\phi)}{a_i}. \end{array} \] Por outro lado, \[ \begin{array}{lll} h_i(\gamma) &\propto& \exp\left\{\int_{\Phi_i(\gamma)} \frac{g_2(\phi)}{a_i}\ln[f_1(\gamma)g_1(\phi)]d\phi\right\} \\ &=& \exp(\ln(f_1(\gamma))\exp\left\{\int_{\Phi_i(\gamma)} a_i^{-1}g_2(\phi)\ln[g_1(\phi)]d\phi\right\} \\ &=& b_if_1(\gamma), \end{array} \] com \(b_i=\exp\left\{\int_{\Phi_i(\gamma)} a_i^{-1}g_2(\phi)\ln[g_1(\phi)]d\phi\right\}\). Como \(a_i\) e \(b_i\) são constantes, \[ \begin{array}{lll} h(\gamma,\phi) &=& \lim_{i\to\infty}\frac{h_i(\phi|\gamma)h_i(\gamma)}{h_i(\phi_0|\gamma_0)h_i(\gamma_0)} \\ &=& \lim_{i\to\infty}\frac{a_i^{-1}g_2(\phi)b_if_1(\gamma)}{a_i^{-1}g_2(\phi_0)b_if_1(\gamma_0)} \propto f_1(\gamma)g_2(\phi). \end{array} \]


Capítulo 3

3.1

  1. Para \(0<x<1\), \(p(x)=1/3\) e \(h(\theta_1,\theta_2|x)= 6(\theta_2-\theta_1)^{-4},\ \theta_1<0, \theta_2>1.\)

3.2

  1. \(p(y|\{x_i\})=\frac{B(A+m,B+y)}{(m+y)B(m,y+1)B(A,B)}, A=a+mn, B=b+\sum_i x_i\)

3.3

  1. \(Bi(N-1,p)\) é a distribuição a posteriori de \(\theta-1\) ou de \(\theta\) consoante \(x\) indica macho ou fêmea, respetivamente.
  2. \(E_X\left[h(\theta|X)\right]=h(\theta).\)

3.4

\(h(N) \propto 1/N,\ N=1,2,\ldots \rightarrow h(N|x)= x/N^2,\ N=x,x+1,\ldots .\)
Moda a posteriori \(= 100 = EMV = 1/2 \times\,\) mediana a posteriori
(Nota: Média a posteriori não finita).

3.6

  1. \(h(M) = 1/(N+1) \rightarrow h(M|x)=\frac{{M \choose x}{N-M \choose n-x}}{{N+1 \choose n+1}}\).
  2. \(\frac{x+1}{n+2}.\)

3.7

  1. Sugestão: Prove para cada componente de \(\alpha\) que $E_h(_j|x)= $, onde \(x_j\) é a correspondente componente de \(x\).
  1. Sugestão: Tenha em conta que \(\mu(\alpha)=\frac{\partial \psi(\alpha)}{\partial \alpha}.\)

3.10

  1. IC HPD: \(\left[x_1, x_1 \sqrt{(1-\gamma)^{-1}}\right]\).
  2. 1/3.

3.11

  1. Sugestão: Considere, e.g., \(\bar{x}=a\) e \(c \rightarrow \infty.\)

3.12

Use as transformações \(y=x-c\) em \(1-F_{Ga(a,b)}(c)\) e \(y=(c-x)/c\) em \(F_{Be(a,b)}(c)\).

3.14

\(n=59\).

3.15

\(n=10,\,\bar{x}=1.6\).

3.16

  1. \(h(\eta |x_1,\ldots,x_n)\) imprópria, \(\forall n\)\(h(\sigma^2 |x_1,\ldots,x_n)\) própria \(\forall n>1\) e imprópria para \(n=1\)\(h(\mu |x_1,\ldots,x_n)\) própria, \(\forall n\). O uso em modelos amostrais inidentificáveis de distribuições a priori impróprias conduz a distribuições a posteriori também impróprias.
  2. \(\mu|\alpha,x \sim N\left(\bar{\mu}_{\alpha}(x), \frac{b^2 \sigma_0^2/n}{b^2 + \sigma_0^2/n}\right), \ \bar{\mu}_{\alpha}(x)=\frac{\frac{a+\alpha}{b^2}+\frac{n \bar{x}}{\sigma_0^2}}{\frac{1}{b^2}+\frac{n}{\sigma_0^2}};\)
    \(\alpha|x \sim N\left(\bar{\alpha}(x), \frac{d^2(b^2 +\sigma_0^2/n)}{d^2 + b^2 + \sigma_0^2/n}\right);\)
    \(\alpha|\mu,x \stackrel{\rm d}{\equiv} \alpha |\mu \sim N\left(\bar{\alpha}_\mu, \frac{b^2 + d^2}{b^2 d^2}\right),\ \bar{\alpha}_\mu=\frac{b^2c+d^2(\mu-a)}{b^2+d^2}\);
    \(\mu|x \sim N\left(\bar{\mu}(x), \frac{(b^2+d^2) \sigma_0^2/n}{b^2 + d^2 +\sigma_0^2/n}\right), \ \bar{\mu}(x)=\frac{\frac{a+c}{b^2+d^2}+\frac{n \bar{x}}{\sigma_0^2}}{\frac{1}{b^2+d^2}+\frac{n}{\sigma_0^2}}.\)
    Atualização previsível das distribuições a priori, marginal e condicional, de \(\mu\) e consequentemente da distribuição a priori marginal de \(\alpha\), contrariamente à distribuição a priori condicional de \(\alpha\) devido à inidentificabilidade do modelo.

3.17

  1. Nota: \(\widehat{\theta}=1/\bar{x},\, \widehat{\lambda}=\left[n^{-1}\sum_i(x_i^{-1}-\bar{x}^{-1})\right]^{-1}\)
    IC HPD para \(\theta:\begin{cases} (\frac{1}{\bar{x}}\pm c(\gamma)),& \gamma:c(\gamma) \leq 1/\bar{x}\\ (0,\frac{1}{\bar{x}}\pm c(\gamma)),& c.c. \end{cases}\).

3.18

  1. A condição de Savage não é verificada.

3.21

  1. 1/2.
  2. 1/4.

3.22

\(B(x)=\frac{b\sqrt{2}}{B_{\cdot}} e^{-(A_1-A_2)^2/(2 B_{\cdot}^2})\), com \(B_{\cdot}^2= \sum_{i=1}^2 \left(\frac{1}{b^2}+\frac{n_i}{\sigma^2}\right)^{-1}\) e \(A_i=\frac{\frac{a}{b^2}+\frac{n_i \bar{x}_i}{\sigma^2}}{\frac{1}{b^2}+\frac{n_i}{\sigma^2}}\), \(i=1,2.\)

3.23

Modelo amostral: \(X|n, \alpha \sim Bi(n, \alpha) \amalg Y|m, \beta \sim Bi(m, \beta)\).
Distribuição a priori possível compatível com \(0 < \alpha \leq \beta <1\): \(\beta \sim Be(b_1, b_2); \phi= \alpha/\beta \sim Be(a_1, a_2)\) com hiperparâmetros eliciados a partir de crenças a priori sobre, e.g., médias e desvios padrões de \(\beta\) e \(\phi\).
Distribuição a posteriori conjunta de \(\beta\) e \(\phi\): mistura finita do produto de duas densidades a posteriori Beta.
\(P(\phi \leq u|x,y)\) expressável como mistura finita de funções de distribuição a posteriori Beta.

3.24

Sugestões: i) \(Z=X_1 - X_2,\ f_Z(z)= {\frac{1}{2}} e^{-z} I_{\left[0,+\infty \right)}(z)+ {\frac{1}{2}} e^{z} I_{\left(-\infty,0 \right)}(z)\); ii) \(W=X_1/X_2, \ f_W(w)= (1+w)^{-2} I_{\left(0,+\infty \right)}(w).\)

3.25

Os fatores de Bayes numa e noutra parametrização são idênticos (a \(B(x)=\frac{h_1(\gamma_0|x)}{h_1(\gamma_0)}\) como consequência da condição em a).

3.26

  1. \(h_1(\gamma)= \frac{b_1^{a_1} b_2^{a_2} \gamma^{a_1-1}}{B(a_1,a_2) (b_1 \gamma+b_2)^{a_{\cdot}}}\)\(\gamma >0\)
    \(h_1(\phi|\gamma) \sim Ga(a_{\cdot},\frac{b_1 \gamma + b_2}{1+\gamma})\).
    Admitindo a condição de Savage \(h_0(\phi) = h_1(\phi|\gamma=1)\),
    \(B(t_1,t_2) = \frac {B(a_1,a_2)}{B(A_1,A_2)} \frac{B_1^{A_1} B_2^{A_2}}{b_1^{a_1} b_2^{a_2}} \frac{b_{\cdot}^{a_{\cdot}}}{B_{\cdot}^{A_{\cdot}}}\)
    com \(t_1=\sum_i x_i\), \(t_2=\sum_i y_i\), \(A_i = a_i+t_i\), \(B_i=b_i+n\), \(i=1,2\).

  2. \(B_C(t_1,t_2) = \frac{f_{Bi (t,1/2)}(t_1)}{E_{\gamma}[f_{Bi (t,\frac{\gamma}{\gamma+1})}(t_1)]} \not= B(t_1,t_2)\), em geral, verificando-se a igualdade quando \(b_1=b_2\) com \(B(t_1,t_2) = (\frac{1}{2})^{t_1+t_2} \frac{B(a_1,a_2)}{B(A_1,A_2)}\). Esta identidade justifica-se atendendo a que a estatística \(T=T_1+T_2\) é ancilar parcial para \(\gamma\) e ao respeito dos métodos bayesianos pelo PCG quando a priori \(\gamma \amalg \phi\) (o que garante a sua independência a posteriori), condição que decorre da situação \(b_1=b_2\).

3.27

\(B(t_1,t_2)\) idêntico ao de 3.26 no caso i) e diferente no caso ii) como consequência da condição em 3.25 a) ser satisfeita no 1º caso e violada no 2º.


Capítulo 4

4.1

  1. Aplique o resultado sobre o comportamento assintótico da distribuição a posteriori conjunta de \(\lambda=\mu_1/\mu_2\) e \(\phi=\mu_2\) em termos das estimativas MV referido no Cap. 5.

4.2

Admitindo que as diferenças das observações emparelhadas constituem uma a.a. de um modelo Normal, a distribuição a posteriori de \(\mu_1-\mu_2\) evidencia que as notas pelo método tradicional tendem a ser menores do que as obtidas pelo método personalizado, ainda que as diferenças não sejam estatisticamente suficientes para invalidar a hipótese de um rendimento médio igual (\(P \approx 0.13\)). A conclusão é idêntica se se admitir o modelo configurado por a.a. independentes de distribuições Normais homocedásticas.

4.3

  1. (-4.4214, 0.9614).
  2. (-4.2638, 0.8036), com grau de credibilidade 0.938.

4.5

  1. O menor IC HPD contendo \(\psi \equiv \sigma_1^2/\sigma_2^2=1\), \(1 \leq \psi \leq 5.134\), apresenta um GC 45.3%, aprox..
  2. Sob \(\psi=1\), o nível de plausibilidade relativa a posteriori de \(\mu_1-\mu_2=0\) é \(P \approx 0.20\).

4.7

  1. Sim.
  2. (173.95, 179.35).
  3. (168.52, 184,78).

4.8

  1. Evidência a favor da igualdade de variâncias (\(P \approx 0.505\)) e contra a igualdade das médias no sentido de um maior valor para C (\(P \approx 0\)).
  2. Evidência forte (\(P \approx 0\)) contra a igualdade das 4 médias, a que não é alheio o resultado de diferenças significativas entre a média de A (ou de B) e cada uma das médias referentes a C e D.

Capítulo 5

5.1

  1. \(N(\frac{A-1}{B},\frac{A-1}{B^2})\), \(A=a+ \sum_{i=1}^n x_i>1, B=b+n.\)
  2. Distr. aprox.: \(\phi \sim N(\ln \frac{A}{B},\frac{1}{A}); \psi \sim N(\left[\frac{A-1/2}{B}\right]^{1/2},\frac{1}{4 B}).\)

5.2

\(E(\lambda|\{x_i\})= \frac{A}{B} \left(\frac{A}{A-1}\right)^{1/2} e^{-1}.\)

5.3

Seja \(\theta=(\theta_1,...,\theta_k)\). Considerando o desenvolvimento em série de Taylor até à segunda ordem de \(\psi(\theta)\) em torno de \(\hat\theta\), máximo de \(-\psi(\theta)\) \[ \psi(\theta)\approx \psi(\hat\theta)+\nabla\psi(\hat\theta)(\theta-\hat\theta)+\frac{1}{2}(\theta-\hat\theta)^T\nabla^2\psi(\hat\theta)(\theta-\hat\theta), \] tem-se \[ I\approx f(\hat\theta) \exp\{-n\psi(\hat\theta)\}\int \exp\{-\frac{n}{2}(\theta-\hat\theta)^T\nabla^2\psi(\hat\theta)(\theta-\hat\theta)\} d\theta. \] O resultado sai de imediato considerando que \[ \int \exp\{-\frac{n}{2}(\theta-\hat\theta)^T\nabla^2\psi(\hat\theta)(\theta-\hat\theta)\} d\theta=(2\pi)^{k/2}det(n\nabla^2\psi(\hat\theta))^{-1/2}. \]

5.5

  1. Sugestão: \(E(\psi|x_1, x_2)= E(\theta_1|x_1)E(\theta_2^{-1}|x_2)= E(\theta_1|x_1) \frac{n_2}{x_2-1/2}\), se \(x_2 \geq 1.\)
  2. Pontos de máximo: \(\widehat{\theta}=(\widehat{\theta}_i, i=1,2),\) \(\widehat{\theta}_i= \frac{x_i-1/2}{n_i-1},\) se \(0<x_i<n_i;\)
    \(\theta^{\ast}=(\theta^{\ast}_i, i=1,2)=(\frac{x_1+1/2}{n_1}, \frac{x_2-3/2}{n_2-2}),\) se \(0<x_1<n_1 \cap 2 \leq x_2<n_2.\)
  3. \(\lambda_i, i=1,2 \underset{iid}{\sim} U(0,\pi/2);\) Pontos de máximo:
    \(\widehat{\lambda}=(\widehat{\lambda}_i, i=1,2),\ \widehat{\lambda}_i= arc sen \sqrt \frac{x_i}{n_i};\)
    \(\lambda^{\ast}=(\lambda^{\ast}_i, i=1,2)=(arc sen \sqrt \frac{x_1+1}{n_1+1}, arc sen \sqrt \frac{x_2-1}{n_2-1}).\)

5.6

  1. Pode mostrar-se, usando o método de Berger-Bernardo, que a distribuição a priori de referência quando se considera \(\delta\) o parâmetro de interesse é \[ h(\gamma,\delta)\propto \frac{1}{\gamma\delta}. \] Esta distribuição pode também ser vista como resultante da regra de Jeffreys quando se admite \(\gamma\) e \(\delta\) independentes.
    Consequentemente \[ h(\gamma,\delta|x)\propto \delta^{n-1}\mbox{e}^{-\sum_{i=1}^n\ln(1+\delta x_i)}\gamma^{n-1}\mbox{e}^{-\gamma \sum_{i=1}^n\ln(1+\delta x_i)}, \] daí o resultado indicado.
  2. Usando o facto de que \((\gamma|\delta, x) \sim Ga(n, \sum_{i=1}^n \ln(1+\delta x_i))\), tem-se \[ h(\delta|x)=\frac{\delta^{n-1}\mbox{e}^{-\sum_{i=1}^n\ln(1+\delta x_i)}\left(\sum_{i=1}^n\ln(1+\delta x_i)\right)^{-n}} {\int_0^{+\infty}\delta^{n-1}\mbox{e}^{-\sum_{i=1}^n\ln(1+\delta x_i)}\left(\sum_{i=1}^n\ln(1+\delta x_i)\right)^{-n}d\delta}. \] Aplique-se para o propósito mencionado o método de Laplace ao denominador escrito na forma \[ I=\int_0^{+\infty}f(\delta) \exp\left[-n\psi(\delta)\right]\,d\delta, \] com \(f(\delta)=\delta^{-1}\) e \(\psi(\delta)=\frac{\sum_{i=1}^n\ln(1+\delta x_i)}{n}+\ln\left(\frac{\sum_{i=1}^n\ln(1+\delta x_i)}{\delta}\right).\)
  3. O resultado aparece por aplicação do método de Laplace ao numerador e denominador de \[ h(\delta|x)=\frac{\int_0^{+\infty}\delta^{n-1}\mbox{e}^{-\sum_{i=1}^n\ln(1+\delta x_i)}\gamma^{n-1}\mbox{e}^{-\gamma \sum_{i=1}^n\ln(1+\delta x_i)}d\gamma}{\int_0^{+\infty}\int_0^{+\infty}\delta^{n-1}\mbox{e}^{-\sum_{i=1}^n\ln(1+\delta x_i)}\gamma^{n-1}\mbox{e}^{-\gamma \sum_{i=1}^n\ln(1+\delta x_i)}d\gamma d\delta}, \] usando a expressão (5.16) para o numerador, considerando \(\delta\) fixo, e a expressão do Exerc. 5.3 para o denominador. Note-se que a aproximação de Tanner é equivalente à expressão (5.26).
    ATENÇÃO: Veja a errata sobre o \(n\) espúrio no denominador de (5.26).
  4. Segundo a aproximação em b):
    Um gráfico de valores de \(\delta\) contra \(\psi^{\prime}(\delta)\) mostra que existe uma raiz no intervalo (0,2). A raiz é aproximadamente 0.23326. Portanto, \(\hat{\delta}\approx 0.23326\), \(\psi(\hat{\delta})\approx 4.0052\) e \(\psi^{\prime\prime}(\delta)\approx 0.3839\).
    Segundo a aproximação em c):
    Denominador: O par \((\gamma,\delta)\) que maximiza \(h(\gamma,\delta)f(x|\gamma,\delta)\) é aproximadamente \((5.4252,0.1971)\) e \(\det(\hat\Sigma) \approx 0.0259.\)
    Numerador: para \(\delta\) fixo, \(\hat\gamma^*=\frac{n-1}{\sum_{i=1}^n \ln(1+\delta x_i)}\) e \(\hat\Sigma^*=\frac{n-1}{\left(\sum_{i=1}^n \ln(1+\delta x_i)\right)^{2}}.\)
    Um gráfico de \(h(\delta|x)\) usando as duas aproximações mostra um bom acordo entre elas.

5.7

  1. É conveniente, por razões computacionais, centrar os valores da covariável, isto é usar o preditor logístico \[ \ln\left(\frac{\theta_i}{1-\theta_i}\right)=\alpha^* + \beta (x_i-\overline{x}), \] e considerar como parâmetros do modelo \((\alpha^*,\beta)\), sendo \(\alpha=\alpha^*-\beta\overline{x}\). Dado que \((\alpha^*,\beta)\in \mathbb{R}^2\), uma distribuição a priori não informativa para \((\alpha^*,\beta)\) é \(h(\alpha^*,\beta)\propto 1.\) Consequentemente, a respetiva distribuição a posteriori é proporcional a \[ h(\alpha^*,\beta|x,n,y)\propto \exp\left\{\sum_{i=1}^Ny_i(\alpha^*+\beta(x_i-\overline{x}))-\sum_{i=1}^Nn_i\ln(1+\mbox{e}^{\alpha^*+\beta(x_i-\overline{x})})\right\}, \] e \[ h(\beta|x,n,y)=\frac{\int_{-\infty}^{+\infty}h(\alpha^*,\beta|x,n,y)d\alpha^*} {\int_{-\infty}^{+\infty}\int_{-\infty}^{+\infty}h(\alpha^*,\beta|x,n,y)d\alpha^* d\beta}, \] com \(x=(x_1,...,x_N), n=(n_1,...,n_N), y=(y_1,...,y_N)\) e \(N\) a dimensão da amostra.
    Vai exemplificar-se como método de integração numérica a regra dos trapézios. Para o efeito sejam \([\alpha^*_I,\alpha^*_S]\) e \([\beta_I,\beta_S]\) intervalos suficientemente amplos para os valores de \(\alpha^*\) e \(\beta\), respetivamente. Considere-se uma partição desses intervalos da forma \([\alpha_I,\alpha_I+h_\alpha,...,\alpha_S=(n_\alpha-1)h_\alpha]\) e \([\beta_I,\beta_I+h_\beta,...,\beta_S=(n_\beta-1)h_\beta]\). Para cada \(\beta\) obtém-se pela regra dos trapézios uma aproximação para \(\int_{-\infty}^{+\infty}h(\alpha^*,\beta|x,y,n)d\alpha^*\), obtendo-se assim valores tabelados do numerador de \(h(\beta|x,y,n)\). Designando esses valores por \(F_i,i=1,...,n_\beta\), o integral do denominador é, pela regra dos trapézios igual a \(h_\beta(\frac{F_1+F_{n_\beta}}{2}+\sum_{i=2}^{n_\beta-1} F_i).\)
  2. Sejam \((\hat{\alpha^*},\hat{\beta})\) os valores de \((\alpha^*,\beta)\) que maximizam \[ \sum_{i=1}^Ny_i(\alpha^*+\beta(x_i-\overline{x}))-\sum_{i=1}^Nn_i\ln(1+\mbox{e}^{\alpha^*+\beta(x_i-\overline{x})}) \] e \[ \hat{\Sigma}=\left( \begin{array}{cc} \hat{\sigma}^2_{\alpha} &\hat{\sigma}_{\alpha,\beta }\\ \hat{\sigma}_{\alpha,\beta } & \hat{\sigma}^2_{\beta} \\ \end{array} \right), \] a inversa do negativo da matriz das suas segundas derivadas calculadas em \((\hat{\alpha^*},\hat{\beta})\). A distribuição marginal a posteriori de \(\beta\) é \(N(\hat{\beta},\hat{\sigma}^2_{\beta})\), admitindo a aproximação normal bivariada para a distribuição a posteriori \(h(\alpha^*,\beta|x,n,y)\).
    A aproximação pelo método de Laplace obtém-se usando a expressão (5.26).
  3. Por aplicação aos dados da tabela apresentada tem-se \[ (\hat{\alpha^*},\hat{\beta})\approx (0.7438 , 34.2703), \quad \hat{\Sigma}=\left( \begin{array}{cc} 0.0190 & 0.1271 \\ 0.1271 & 8.4805 \\ \end{array} \right). \] Para a integração numérica pode considerar-se, por exemplo, \(\alpha^*_I=0.2, \alpha^*_S=2, h_\alpha=0.001, \beta_I=20, \beta_S=50, h_\beta=0.01\).
    Para o método de Laplace pode usar-se uma divisão mais fina do intervalo para \(\beta\) para obter valores tabelados do numerador da expressão (5.26). Para cada \(\beta\) fixo obtém-se o valor de \(\alpha^*\) que maximiza \[ \sum_{i=1}^Ny_i(\alpha^*+\beta(x_i-\overline{x}))-\sum_{i=1}^Nn_i\ln(1+\mbox{e}^{\alpha^*+\beta(x_i-\overline{x})}). \] Seja ele \(\hat{\alpha}^{\star}_{\beta}\). Para cada par \((\beta,\hat{\alpha}^*_\beta)\) obtém-se o correspondente valor para o negativo do inverso da segunda derivada daquela expressão em relação a \(\alpha^*\). O cálculo do denominador é imediato.

5.8

  1. Facilmente se prova que \(Y=\min(T_1,T_2,T_3)\sim Exp(\alpha\exp(\beta x))\) com \(\alpha=\alpha_1+\alpha_2+\alpha_3\). A probabilidade de morrer devido à causa \(C_i\) é \(P(T_i\leq Z)\), com \(Z=\min(T_j,T_k)\sim Exp((\alpha_j+\alpha_k)\exp(\beta x))\), \(j,k\neq i\). \[ P(T_i\leq Z)=\int_0^\infty\left[\int_0^z f_i(t_i|x,\alpha_i,\beta)dt_i\right] (\alpha_j+\alpha_k)\exp(\beta x-(\alpha_j+\alpha_k)z\mbox{e}^{\beta x})dz=\frac{\alpha_i}{\alpha}. \]
  2. Calculando para o efeito \(P(Y\leq y, I=i)\) tem-se \[ \begin{array}{lll} P(Y\leq y, I=i)&=&P(T_i\leq y,T_j>T_i,T_k>T_i) \\ &=& \int_0^y\left[\int_{t_i}^\infty\int_{t_i}^\infty \prod_{\ell=1}^3 f_\ell(t_\ell|\alpha_\ell,\beta)dt_jdt_k\right]dt_i\\ &=& \int_0^y\alpha_i\mbox{e}^{\beta x }\exp(-\alpha_i\mbox{e}^{\beta x }t_i)(\exp\left\{-(\alpha_j+\alpha_k)\mbox{e}^{\beta x }t_i\right\})dt_i\\ &=& \frac{\alpha_i}{\alpha} \left(1-\mbox{e}^{-\alpha y\mbox{e}^{\beta x}}\right). \end{array} \] Derivando em ordem a \(y\) obtém-se o resultado pretendido.
  3. Tenha em conta b).
  4. Siga a sugestão referida.
  5. Seja \(\textbf{n}=(n_i), \textbf{x}=(x_{ij}), \textbf{y}=(y_{ij}), i=1,2,3; j=1,...,n_i\). Facilmente se reconhece que \[ h(\psi,\alpha,\gamma|\textbf{n},\textbf{x},\textbf{y})=h(\psi|\textbf{n})h(\alpha|\textbf{n},\textbf{x},\textbf{y},\gamma)h(\gamma|\textbf{n},\textbf{x},\textbf{y}), \] com \[ \begin{array}{lll} h(\psi|\textbf{n}) &=& D_2(G_1,G_2,G_3) \\ h(\alpha|\textbf{n},\textbf{x},\textbf{y},\gamma) &=& Ga(G,\gamma H_\gamma)\\ h(\gamma|\textbf{n},\textbf{x},\textbf{y}) &\propto& \gamma^{S-1}\mbox{e}^{-\gamma } H_\gamma^{-G} . \end{array} \] Seja \(Y^*= min(T_1, T_2, T_3)\) o tempo de vida de um futuro indivíduo com covariável \(x^*\). A probabilidade preditiva de morrer antes de \(y\) devido à causa \(C_i\) é dada por \[ P(I=i,Y^*\leq y|x^*,\textbf{n},\textbf{x},\textbf{y})= \int_0^y\int\int\int\alpha_i\exp\{\beta x-\alpha t \mbox{e}^{\beta x}\}h(\psi,\alpha,\gamma|\textbf{n},\textbf{x},\textbf{y})d\psi d\alpha d\gamma dt. \] O resultado apresentado sai de imediato após algum cálculo algébrico.
  6. Sugestão: use uma distribuição a priori não informativa para os vários parâmetros do modelo e considere o cálculo da probabilidade preditiva para alguns valores específicos de \(y\).

5.9

Fazendo a transformação \(y=t/x\) tem-se \[ \begin{array}{lll} \Gamma(x+1) &=& \int_0^{+\infty} (xy)^x\exp(-xy)xdy \\ &=& x^{x+1}\int_0^{+\infty} y^x\exp(-xy)dy \\ &=& x^{x+1} \int_0^{+\infty}\exp[-x(y-\log(y))]dy \end{array} \] Tomando \(g(y)=y-log(y)\), tem-se \(g^\prime(y) = 1 - \frac{1}{y},\ g^{\prime\prime}(y) = \frac{1}{y^2}\). A função \(g(y)\) atinge um máximo em \(\hat{y}=1\) e \(g^{\prime}(\hat{y})=1, g^{\prime\prime}(\hat{y})=1\).
Assim, aplicando o método de Laplace (veja-se equação (5.16)) ao integral envolvido em \(\Gamma(x+1)\), obtém-se \[ \begin{array}{lll} \Gamma(x+1) &=& x^{x+1} \sqrt{2\pi}x^{-1/2}\exp(-x)\left\{1+O\left(\frac{1}{x}\right)\right\} \\ &=& \sqrt{2\pi}x^{x+1/2}\exp(-x)\left\{1+O\left(\frac{1}{x}\right)\right\} \end{array} \]

5.10

  1. A função dada pode escrever-se na forma \[ f(x|\psi)\propto \prod_{i=1}^{n-1} \psi^{1/2}\exp\left(-\frac{\psi}{2}(x_{i+1}-x_i)^2\right), \] que corresponde ao produto de \(n-1\) funções densidade de probabilidade normais condicionais com valor médio \(x_i\) e precisão \(\psi\), expresso por \(f(x|\psi)=\prod_{i=1}^{n-1} f(x_{i+1}|x_i,\psi)\).
    Assim, dado \(X_i=x_i\), \(X_{i+1}\) é independente designadamente de \(X_{i-1}\). O grafo que exprime as relações de dependência condicional dos elementos do vetor \(X\) é pois descrito por \[ \mathcal{G}=(\mathcal{V}=\{1,...,n\},\mathcal{E}),\ \ \mathcal{E}=\{\{i,j\}:j=i+1, i=1,...,n-1\}. \]
  2. Como \[ \sum_{i=1}^{n-1}(x_{i+1}-x_i)^2=x_1^2+x_n^2+2\sum_{i=2}^{n-1}x_i^2-2 \sum_{i=1}^{n-1}x_i x_{i+1}, \] a função dada pode ainda escrever-se na forma \[ f(x|\psi)\propto \psi^{(n-1)/2}\exp\left(-\frac{\psi}{2} x^\prime R x\right), \] com \[ R=\left( \begin{array}{ccccccccc} 1 & -1 & 0 &0&0&... & 0&0&0 \\ -1 & 2 & -1 & 0 &0&... & 0&0&0 \\ 0 & -1 & 2 & -1 &0& ...& 0&0&0\\ ...& ... & ... & ... &...&...&...&...&... \\ 0 & 0 & 0 & 0 &0&...& -1 & 2&-1 \\ 0 & 0 & 0 & 0 &0& ... & 0 & -1&1\\ \end{array} \right) \]
    sendo então a matriz de precisão dada por \(Q=\psi R\).
    A soma dos elementos de cada linha de \(R\) é zero, o que significa que a soma das \(n\) colunas é o vetor nulo. A matriz \(R\) não tem assim caraterística completa e, por conseguinte, o modelo considerado para \(X\) é um IGMRF em que \(f(x|\psi)\) corresponde a uma “densidade” de uma distribuição normal multivariada degenerada, com vetor média nulo e matriz de precisão \(\psi R\), com \(R\) não invertível (veja-se alínea seguinte).
  3. A simetria de \(R\) é óbvia da sua representação que mostra que \(R_{ij}=R_{ji}, i \neq j\).
    Como para \(\forall x=(x_1,...,x_n) \neq 0\) se tem \(x^\prime R x=\sum_{i=1}^{n-1}(x_{i+1}-x_i)^2 \geq 0\), \(R\) é semidefinida positiva, mas não é definida positiva pois \(\exists x \neq 0\), \(x^\prime R x=0\) (e.g., \(x=1_n\)).
    A transformação de \(R\) por aplicação sucessiva da operação de substituição da linha \(i\) (começando em \(i=2\)) pela soma dessa linha com a linha que a precede conduz à matriz \(R^*\) abaixo que tem visivelmente caraterística \(n-1\). Como este tipo de operações não muda a caraterística das sucessivas matrizes, fica demonstrada a afirmação em falta. \[ R^*=\left( \begin{array}{ccccccccc} 1 & -1 & 0 &0&0&... & 0&0&0 \\ 0 & 1& -1 & 0 &0&... & 0&0&0 \\ 0 & 0 & 1 & -1 &0& ...& 0&0&0\\ ...& ... & ... & ... &...&...&...&...&... \\ 0 & 0 & 0 & 0 &0&...& 0 & 1&-1 \\ 0 & 0 & 0 & 0 &0& ... & 0 & 0&0\\ \end{array} \right) \]
    1. Caso \(i=1\): \[ \begin{array}{lll} f(x_1|x_2,...,x_n)&=&\frac{\exp\left(-\frac{\psi}{2} \sum_{i=1}^{n-1}(x_{i+1}-x_i)^2\right)}{\int_{-\infty}^{+\infty} \exp\left(-\frac{\psi}{2}(x_{2}-x_1)^2\right)\exp\left(-\frac{\psi}{2}\sum_{i=2}^{n-1}(x_{i+1}-x_i)^2\right)dx_1} \\ &=& \frac{\exp\left(-\frac{\psi}{2}(x_{2}-x_1)^2\right)}{\int_{-\infty}^{+\infty} \exp\left(-\frac{\psi}{2}(x_{2}-x_1)^2\right)dx_1} =\frac{\psi^{\frac{1}{2}}}{\sqrt{2\pi}}\exp\left(-\frac{\psi}{2}(x_1-x_2)^2\right) \end{array} \]
    1. Caso \(i=n\): Raciocínio semelhante escrevendo \[\sum_{i=1}^{n-1}(x_{i+1}-x_i)^2=\sum_{i=1}^{n-2}(x_{i+1}-x_i)^2+(x_{n}-x_{n-1})^2\]
    1. Caso \(1<i<n\): Decomposição \[\sum_{i=1}^{n-1}(x_{i+1}-x_i)^2=\sum_{j=1}^{i-2}(x_{j+1}-x_j)^2+\sum_{j=i-1}^{i}(x_{j+1}-x_j)^2+ \sum_{j=i+1}^{n-1}(x_{j+1}-x_j)^2,\] com o termo do meio do 2membro expresso por (aplicação da identidade algébrica exibida no Exemplo 3.2) \[\sum_{j=i-1}^{i}(x_{j+1}-x_j)^2 = 2(x_i-c_i)^2 + \frac{1}{2}(x_{i+1}-x_{i-1})^2,\quad c_i= (x_{i+1}+x_{i-1})/2,\] ficando por cancelamento de fatores comuns no numerador e denominador \[ \begin{array}{lll} f(x_i|x_{-i}) &=& \frac{\exp\left[-\frac{\psi}{2}2(x_i-c_i)^2\right]} {\int_{-\infty}^{+\infty} \exp\left[-\frac{2\psi}{2}(x_i-c_i)^2\right]dx_i} \\ &=&\frac{(2\psi)^{\frac{1}{2}}}{\sqrt{2\pi}}\exp\{-\frac{2\psi}{2}\left[x_i-(x_{i+1}+x_{i-1})/2)\right]^2\} . \end{array} \]

5.11

  1. Como os incrementos de segunda ordem \(\Delta^2 x_i=x_{i+2}-2x_{i+1}-x_i\) são i.i.d. \(N(0,\psi^{-1})\), então a f.d.p. (degenerada) do vetor \(X\) (veja-se uma justificação adequada no capítulo 3 de Rue e Held, 2005) é proporcional a \[ \begin{array}{lll} f(x|\psi)&\propto& \prod_{i=1}^{n-2} \psi^{1/2} \exp\{-\frac{\psi}{2}\Delta_i^2\}\\ &=&\psi^{n-2/2}\exp\{-\frac{\psi}{2}\sum_{i=1}^{n-2}(x_{i+2}-2x_{i+1}-x_i^2)\} \end{array} \]
  2. Verifique-se que \(\sum_{i=1}^{n-2}\Delta_i^2=(Dx)^T(Dx)\) com \(D\) matriz \((n-2)\times n\) da forma \[ D=\left( \begin{array}{ccccccc} 1 &-2 & 1& & & & \\ & 1 & -2 & 1 & & & \\ & & 1 & -2& 1 & & \\ & & \ddots & \ddots & \ddots & & \\ & & & & 1 & -2 & 1 \\ \end{array} \right). \]
    Sendo assim, \[ f(x|\psi) \propto \psi^{(n-2)/2}\exp\{-\frac{\psi}{2}x^T(D^TD)x\} =\psi^{(n-2)/2}\exp\{-\frac{\psi}{2}x^TRx\}, \]
    com a matriz \(R=D^TD\) da forma \[ R=\left( \begin{array}{ccccccccc} 1 & -2 & 1 & & & & & & \\ -2& 5 & -4 & 1 & & & & & \\ 1 & -4 & 6 & -4 & 1 & & & & \\ & 1 & -4 & 6 & -4 & 1 & & & \\ & & & \ddots & \ddots & \ddots & & & \\ & & & 1 & -4 & 6 & -4 & 1 & \\ & & & & 1 & -4 & 6 & -4 & 1 \\ & & & & & 1 & -4 & 5 & -2 \\ & & & & & & 1 & -2 & 1 \\ \end{array} \right). \]
    Para verificar experimentalmente que \(R\) tem caraterística \(n-2\), pode-se considerar o caso de \(n=8\), por exemplo, e usando a função eigen() do software R, obter os 8 valores próprios
    \(\lambda=( 14.573, 10.920, 6.547, 2.956, 0.881, 0.124,0,0)\).
    Como \(R\) tem dois valores próprios iguais a zero, a sua caraterística é 6.
  3. Note-se que a soma dos elementos de cada linha de R é zero e a soma dos elementos de cada linha após multiplicados pelos inteiros respetivamente de 1 a \(n\) também é zero. Ou seja, \(\sum_j R_{ij}=0\) e \(\sum_j j R_{ij}=0\) para cada \(i\) em conjunto traduzem precisamente que \(R\times S=0_{(n,2)}\). Compactamente, aquelas equações exprimem-se pelas seguintes duas restrições nos vetores-coluna \(c_j\) de \(R\), \(\sum_j c_j=0_{(n,1)}\) e \(\sum_j j c_j=0_{(n,1)}\), o que confirma que o número de colunas linearmente independentes (i.e., caraterística) de \(R\) é \(n-2\).

5.12

  1. Do grafo da Figura 5.4 tem-se que \({\cal{E}} =\{\{1,2 \},\{ 2,3 \},\{ 2, 4\}\{ 2,5 \}\}\), e portanto \[ f(x|\tau) \propto \tau^{(5-1)/2}\exp\left(-\frac{\tau}{2}\sum_{j=1,j\neq 2}^5(x_2-x_j)^2\right). \] A soma dos quadrados das diferenças das \(n=5\) componentes de \(x\) pode exprimir-se pelo produto interno entre o vetor das 4 diferenças \(x_2-x_j\), definido por \(D x\), e o seu transposto, em que \(D\) está explicitado abaixo. Assim, \[\sum_{j=1,j\neq 2}^5(x_2-x_j)^2 = x'D'Dx \equiv x'Rx\] em que \[ D=\left( \begin{array}{ccccc} -1 & 1 & 0 & 0 & 0 \\ 0 & 1 &-1 & 0 & 0 \\ 0 & 1 & 0 &-1 & 0 \\ 0 & 1 & 0 & 0 &-1 \\ \end{array} \right),\qquad R=\left( \begin{array}{ccccc} 1 & -1 & 0 & 0 & 0 \\ -1 & 4 & -1 & -1 &-1 \\ 0 & -1 & 1 & 0 & 0 \\ 0 & -1 & 0 & 1 & 0 \\ 0 & -1 & 0 & 0 & 1 \\ \end{array} \right), \]
    Consequentemente, a matriz de precisão é \(Q=\tau R\), estando os seus elementos de acordo com o indicado no enunciado dado que o vértice 2 tem 4 vizinhos (1,3,4,5) e os restantes têm 1 vizinho (que é o 2).
  2. É evidente que a soma dos elementos de cada linha de \(R\) é nula, isto é, o produto de \(R\) pelo vetor unitário \(R \times 1_5=0\) é um vetor nulo. Isto traduz que há uma restrição entre as 5 colunas \(c_j\) dada por \(c_2=-c_1-c_3-c_4-c_5\), implicando que a caraterística de \(R\) é no máximo de 4. Através de operações elementares sobre linhas ou colunas de \(R\), que não alteram a caraterística, é fácil obter um menor de ordem 4 não nulo (, a submatriz \(I_4\)), o que prova que a caraterística de \(R\) é 5-1 e, consequentemente, a natureza de $ f(x|)$ como uma densidade imprópria.
    Uma alternativa de prova da singularidade de \(R\) é calcular os seus valores próprios \(\lambda\) resolvendo a equação \(|R-\lambda I_5|=0\), cujas raízes são \(\lambda_1=5,\lambda_2=\lambda_3=\lambda_4=1,\lambda_5=0\). O facto de haver um valor próprio nulo mostra que a caraterística é 4.
  3. Usando \[ f(x_i|x_{-i},\tau)=\frac{f(x|\tau)}{\int_{-\infty}^{+\infty}f(x|\tau)dx_i}, \] é fácil mostrar que \[ \begin{array}{lll} f(x_j|x_{-j} )&\sim& N(x_2,\frac{1}{\tau}), j=1,3,4,5 \\ f(x_2|x_{-2} ) &\sim& N\left(\frac{1}{4}\sum_{i: i\sim 2}x_i,\frac{1}{4\tau}\right) \end{array} \] (aplique-se repetidamente a fórmula da soma de duas formas quadráticas com um termo comum).

Capítulo 6

6.1

  1. \(n|N, \{\mu_i\} \sim M_{c-1}(N, \theta)\). Dado \(N\), com \(A \equiv \sum_{i=1}^{c}{A_i}=\sum_{i=1}^{c}(a_i+n_i)\) e \(B=b+1\), \(\sum_{j=1}^{c}\mu_j \sim Ga(A,B) \amalg (\theta_1, \ldots, \theta_{c-1}) \sim D_{c-1}(A_1, \ldots, A_{c-1}, A_{c}).\)

6.2

Considere a transformação \((\gamma_1, \ldots, \gamma_c) \leftrightarrow (\theta_1, \ldots, \theta_{c-1},\sum_{i=1}^{c} \gamma_i).\)

6.3

  1. Determine-se a fgm de \(M=Z'n\) à custa da de \(n\). Por escolha apropriada de \(Z\) obtêm-se as distribuições marginais, de qualquer dimensão, de componentes de \(n\).
  2. Aplique-se a expressão do valor esperado de uma função do vector aleatório \(\theta\) e tenha em conta a definição da função beta multivariada. Os momentos de 1ª e 2ª ordens obtêm-se por escolha apropriada dos \(\{r_i\}\).
    Considere-se \(\alpha \equiv Z' \theta= \frac{Z' \gamma}{1'_s(Z' \gamma)},\) com \(\gamma=(\gamma_1, \ldots, \gamma_c)\) e o resultado de 6.2. Aplique-se um argumento análogo a cada \(\pi_k\) expressável por \(\pi_k=(\frac{\gamma_i}{\sum_{j \in C_k}^{}{\gamma_j}}, i \in C_k).\)
    As propriedades restantes envolvendo a transformação \((\theta_1, \ldots, \theta_{c-1}) \rightarrow (\alpha_k, k=1,\ldots,s-1, \bar{\pi}_k, k=1,\ldots,s)\) decorrem da relação entre as suas distribuições conjuntas, tendo em conta que o jacobiano daquela transformação é \(\prod_{k=1}^{s}{\alpha_k^{d_k-1}}.\)
  3. Decorrem de a) e b).

6.4

  1. Mostre-se que a perda esperada preditiva é \(\sum_{u=0,1} L(u,a)f(u|y,z)\), com \(f(u|y,z)= f(u|v)f(y|z,u).\)
  2. Condições: \(k_0=k_1\) e \(f(1|v)=1/2.\)
  3. \(f(u|v)\equiv f(u)=1/2, k_0=k_1\) e \(a=1_c.\)

6.5

  1. Considerando que as unidades amostradas são as numeradas de 1 a n (sem quebra de generalidade), \(x= \sum_{k=1}^{n}{U_k} \amalg \theta-x = \sum_{k=n+1}^{N}{U_k}|\phi\), com \(x|\phi \sim M_{c-1}(n,\phi),\ \theta-x|\phi \sim M_{c-1}(N-n,\phi)\) e \(\phi \sim D_{c-1}(a).\)
  2. \(h(\theta-x|x)=E_{\phi|x}\left[h(\theta-x|\phi)\right]\sim MD(N-n,a+x)\) porque \(\phi|x \sim D_{c-1}(a+x)\).
  3. Tenha-se em conta que \(\theta-x \sim MD(N-n,a), \ x|\theta,N,n \sim Hpg(\theta,N,n)\) e \(\theta-x|x \sim MD(N-n,a+x)\).

6.6

  1. A matriz jacobiana da transformação \((\theta_1, \ldots, \theta_{c-1}) \rightarrow (\phi_1, \ldots,\phi_{c-1})\), com \(\theta_i= \phi_i \prod_{k=1}^{i-1}{(1-\phi_k)}\) e \(1-\sum_{i=1}^{c-1} \theta_i=\prod_{i=1}^{c-1}{(1-\phi_i)}\), é triangular inferior com determinante \(\prod_{i=1}^{c-2}{(1-\phi_i)}^{c-i-1}.\)
  2. Moda conjunta: \(\widetilde{\theta}=(\widetilde{\theta}_i),\ \widetilde{\theta}_i=\frac{a_i-1}{\sum_{j=1}^{c}{a_j-c}}\)
    Modas marginais: \(\widehat{\theta}_i= \frac{a_i-1}{\sum_{j=1}^{c}{a_j-2}} \leq \widetilde{\theta}_i,\ c \geq 2.\)
  3. \(\rho(\theta_i,\theta_j)=-\sqrt \frac{m_i m_j}{(1-m_i)(1-m_j)}\)\(m_i=E(\theta_i)\) e a especificação da matriz de covariância exige apenas mais um hiperparâmetro.
  4. Por transformação para \({\cal S}_{c-1}\) do produto cartesiano de \(c-1\) IC HPD para \(\phi_i\) de GC \(\gamma^{1/c-1}\), obtidos da sua distribuição marginal Beta.

6.7

  1. Use-se sup \(h(\theta)=h(\widetilde{\theta})\), onde \(\widetilde{\theta}\) é a moda da densidade \(h(\theta)\) e o facto de \(\varphi_{\lambda}(t)\) ser o logaritmo natural da f.g.m..
  2. Use-se a expansão assintótica do logaritmo das funções gama para chegar a \[ \varphi_{\lambda}(t) = \alpha_0 - \frac{c-1}{2} \ln (1-2t) + \sum_{k=1}^{\infty} \alpha_k (1-2t)^{-k}, \] onde \(\alpha_0 = - \sum_{k=1}^{\infty} \alpha_k\) e $ _k = .$
    As aproximações \(\chi^2_{(c-1)}\) e \(b\, \chi^2_{(c-1)},\,b=1+B=1-\frac{2 \alpha_1}{c-1},\) implicam a mesma média que resulta de se tomar a expansão de \(\varphi_{\lambda}(t)\) até à ordem \(\{(A_i-1)^{-1}\}\). A aproximação \(d \,\chi^2_{(g)}, \,d=\frac{1+2B}{1+B},\ g=(c-1) \frac{(1+B)^2}{1+2B},\) já apresenta as mesmas média e variância que a referida expansão de \(\varphi_{\lambda}(t)\), com a série reduzida ao seu 1º termo.

6.9

  1. Considere-se os resultados da Subsec. 6.1.1 para o caso de \(c=2, m=1, b_{11}=1, b_{12}=-1\), e tenha-se em conta o Exerc. 6.8.
  2. Considere-se a transformação monótona que relaciona \(\theta\) com \(\ln \frac{\theta}{1-\theta}\). Reveja-se o Exemplo 3.6 e use \(Z=\frac{1}{2}\ln \phi.\)

6.10

  1. \(\beta|n \sim Be(1506, 1314) \overset{a}{\sim} N(0.5341, 8.83 \times 10^{-5}).\)
  1. Forte evidência (\(P=0.99\)) a favor da conjetura de um dado valor para uma apropriada função linear de \(\ln \theta.\)
  1. Não.

6.11

Sugestão: Invoque-se o PSG com base na suficiência parcial de \(n^{\ast}\) para os parâmetros de interesse.

6.12

  1. Calcule-se as dp a priori e a posteriori de \(\gamma^{\ast}\) avaliadas em \(\gamma^{\ast}=0\). Avalie-se as dp a priori de \(\phi\) e as correspondentes verosimilhanças marginais de \(T\).
  2. \(T\) não é ancilar parcial para \(\gamma^{\ast}\) nem \(\phi^{\ast}\) e \(\gamma^{\ast}\) são independentes a priori.

6.14

Siga-se o argumento da Subsec. 6.2.3 partindo do modelo bayesiano Produto de Multinomiais \(\wedge\) Produto de Dirichlet.


Capítulo 7

7.2

Ter em conta que \(\theta=(\theta_0,\theta_1) \in \Theta_1\) e a expressão de \(B(x)\) com \(c_0\) expresso em termos de integral em \(\Theta_1\); provar que \(c_0\) é \(c_1\) multiplicado pelo integral em ordem a \(h_1(\theta|x)\) da razão \(\frac{\bar{h}_0(\theta)}{\bar{h}_1(\theta)}\).

7.8

Mostrar que a função de distribuição de \(\theta^*_n\) avaliada em \(u\), condicional aos \(\{\theta_i\}\), é dada pelo quociente entre \(n^{-1} \sum_{i=1}^n w^*(\theta_i)\, I_{(-\infty,u)}(\theta_i)\) e \(n^{-1} \sum_{j=1}^n w^*(\theta_j)\), com \(w^*(\theta_i) = \frac{\bar{h}(\theta_i|x)}{p(\theta_i)}\), e aplicar a Lei Forte dos Grandes Números para chegar ao limite \(H(u)\). Este limite mantém-se para o valor esperado segundo a função de importância \(p\) daquela função de distribuição condicional de \(\theta^*_n\), invocando o teorema da convergência dominada de Lebesgue.

7.9

  1. A densidade marginal a posteriori de \(\theta^{(m)}\), avaliada em cada ponto fixado.
  2. Aplicar o método SIR.

7.13

  1. Calcular a função de distribuição de \(Y\) condicional a \(U\leq \pi(Y)/[M p(Y)]\).
  2. Ter em conta a) e b).

7.14

Atentar à definição de concavidade pelo sinal da 2ª derivada.

7.15

Ter em atenção eventuais restrições no espaço paramétrico.

7.16

  1. \(F^{-1}(u)= \lambda + \delta tg [\pi(u-1/2)]\);
  2. \(F^{-1}(u) = \delta [-2 \ln (1-u)]^{1/2}\);
  3. \(F^{-1}(u) = \lambda (1-u)^{-1/\delta}\).

7.17

Verificar primeiro que \(f(x,y)= \frac{x}{m(C_{\pi})} I_{A(y)}(x)\), com \(A(y) = (0,\sqrt{\pi(y)})\).


Capítulo 8

8.1

  1. As distribuições a posteriori Normais \(h(\theta_2 | x)\) e \(h(\theta_1 | \theta_2, x)\), por exemplo, dependem ambas de \(x\).
  2. \(\phi_1 | \phi_2,x \sim N(B_1(\phi_2), (\frac{1}{b_1} + \frac{1}{b_2})^{-1})\) com \(B_1(\phi_2) = \frac{\frac{1}{b_1} a_1 + \frac{1}{b_2} (\phi_2-a_2)}{\frac{1}{b_1} + \frac{1}{b_2}}\) coincide previsivelmente com a respetiva distribuição a priori.

8.2

  1. \(\varepsilon_1 = 1 - \frac{\pi_{1\cdot} - \alpha\,\sigma_1}{1-\alpha}\), \(\varepsilon_2 = 1 - \frac{\pi_{1\cdot} \pi_{\cdot 1} - \pi_{11}}{\alpha (\sigma_1 - \pi_{1\cdot})}\), \(\sigma_2 = \frac{ (1-\alpha) \pi_{11} - \pi_{1\cdot} \pi_{\cdot 1} + \alpha \pi_{\cdot 1} \sigma_1}{\alpha (\sigma_1 - \pi_{1\cdot})}\).
  2. O modelo Multinomial trivariado é indexado por 5 parâmetros, \(\theta=(\alpha,\sigma_1,\sigma_2,\varepsilon_1,\varepsilon_2)\); \(g_1(\theta) = (\pi_{11},\pi_{10},\pi_{01})\) e \(g_2(\theta) = (\pi_{11},\pi_{1\cdot},\pi_{\cdot 1})\) são exemplos de funções identificantes; \(\sigma_2\) e \(\varepsilon_2\) não são funções apenas de \(g_2(\theta)\).

8.3

  1. \(\theta \sim Be(c,d|a,b) \ \Leftrightarrow \ h(\theta) = \frac{(\theta-a)^{c-1} (b-\theta)^{d-1}}{B(c,d) (b-a)^{c+d-1}} I_{(a,b)}(\theta) \ \Leftrightarrow \ \theta^{\star} \equiv \frac{\theta-a}{b-a} \sim Be(c,d)\). \(\alpha \sim Unif(0,\frac{1}{2})\) é não informativa quando circunscrita ao suporte imposto [mas já não o é no quadro do suporte padrão \((0,1)\)].
  2. Resposta negativa.
  3. \((\sigma,\varepsilon) \sim Unif(S)\), \(S = \{(\sigma,\varepsilon), 0<\sigma<1, 1-\sigma<\varepsilon<1\}\).

8.4

  1. Note que \(Cov(T_1,T_2) = P(T_1=1,T_2=1) - p_1\,p_2\) com \(p_j = P(T_j=1)\), \(j=1,2\).
  2. \(b_j = p_j\), \(\forall\,j\); o limite superior de \(\mathbb{R}\) é a raiz quadrada da razão das chances de sucesso do melhor versus pior teste; o respetivo limite inferior é o negativo da raiz quadrada do produto das chances de insucesso dos dois testes.

8.5

  1. Denotando \(\theta_j = (p_j,\mu_j)\), \(j=0,1\), \(p_1=p\) e \(p_2=1-p\), \(f(x|\theta_1,\theta_0) = \prod_{i=1}^n \big[ \sum_{j=0}^1 p_j f(x_i|\mu_j) \big]\). Este modelo é inidentificável dado que os ternos \((p,\mu_1,\mu_0)\) e \((1-p,\mu_0,\mu_1)\), \(\forall\,p\in(0,1)\) e \(\mu_j\in \mathbb{R}\), possuem a mesma verosimilhança.
  2. Com \(p\) conhecido, o modelo é identificável se \(p\not=1/2\) e não o é se \(p=1/2\).

8.6

\[ L(\mu,\sigma_1,\sigma_2|{\bf y}) \propto (\sigma_1^2)^{-\frac{I(J-1)}{2}} \exp \{ -\frac{1}{2\,\sigma_1^2} \sum_{i,j} (y_{ij} - \bar{y}_{i\cdot})^2 \} \times (\sigma_1^2 + J \sigma_2^2)^{-\frac{I}{2}} \exp \{ -\frac{J}{2\,(\sigma_1^2 + J \sigma_2^2)} \sum_{i} (\bar{y}_{i\cdot} - \mu)^2) \}. \]

\(\forall\,i,\ \bar{Y}_{i\cdot}|\mu,\sigma_1^2,\sigma_2^2 \ \underset{iid}{\sim} \ N(\mu,\frac{\sigma_1^2 + J \sigma_2^2}{J})\).

\[ h(\sigma_1^2,\Sigma|{\bf y}) \propto (\sigma_1^2)^{-(\frac{I(J-1)}{2}+1)} \exp \{ -\frac{1}{2\,\sigma_1^2} \sum_{i,j} (y_{ij} - \bar{y}_{i\cdot})^2 \} \times (\Sigma)^{-(\frac{I-1}{2}+1)} \exp \{ -\frac{J}{2\,\Sigma} \sum_{i} (\bar{y}_{i\cdot} - \bar{y})^2) \}, \ \sigma_1^2 < \Sigma. \]

8.7

De acordo com o método dos momentos, a EBE da média de \(\theta_i\) é \(\widetilde{\theta}_i = \frac{\widetilde{\tau} \widetilde{\mu}+x_i}{\widetilde{\tau}+m}\), com \(\widetilde{\mu}=\frac{\bar{x}}{m}\),\(\widetilde{\tau} = \frac{s^2_n - m \bar{x}(1-\frac{\bar{x}}{m})}{\bar{x}(1-\frac{\bar{x}}{m}) - s^2_n}\), \(s^2_n = \frac{1}{n} \sum_i (x_i-\bar{x})^2\), \(i=1,\ldots,n\).

8.8

  1. \(\widehat{\theta}_i = \frac{d+x_i}{\widehat{c}}\) com \(\widehat{c}\) dado, respetivamente, por i) \((\frac{1}{n} \sum_i \ln \frac{d+x_i}{d})^{-1}\); ii) \(1+\frac{d}{\bar{x}}\); iii) \((n-1) \{\sum_i \ln \frac{d+x_i}{d} )^{-1}\).
  2. \((d+x_i) E_{c|\{x_i\}}(\frac{1}{c})\).

8.9

\(\bar{x} + \frac{1}{c} (x_i-\bar{x})\), \(i=1,\ldots,n\) (via método dos momentos).

8.10

  1. \(N+N(N-1) \frac{\alpha+1}{c \alpha + 1}\).
  2. \(\frac{\widetilde{\alpha}+n_i}{c \widetilde{\alpha}+N}\), \(i=1,\ldots,n\), com \(\widetilde{\alpha} = \frac{1-T}{cT-1}\), \(T = \frac{\sum_i n_i^2 - N}{N(N-1)}\).

8.11

  1. Usar o valor médio e variãncia da distribuição assintótica de \(\frac{1}{2} W(\theta)\).

Capítulo 9

9.4

M-H com independência:

  1. Distribuição instrumental proposta: \(\beta^{(t+1)} | \beta^{(t)} \sim N_2(\widehat{\beta}, [I(\widehat{\beta})]^{-1})\) com i) \(\widehat{\beta}\) tomado como a média a posteriori de \(\beta\) (ou a respetiva estimativa de MV) obtidas por resolução iterativa do correspondente sistema de equações (a derivar); ii) \(I(\beta)\) tomada com a matriz de informação observada (a derivar).

  2. Rácio de M-H: \(R(u,v) = \frac{\bar{h}(v|y) q(u)}{\bar{h}(u|y) q(v)}\) em que \(\bar{h}\) denota o núcleo da densidade a posteriori de \(\beta\) (expressão a derivar).

M-H com passeio aleatório em bloco:

  1. Distribuição instrumental proposta: \(\beta^{(t+1)} | \beta^{(t)} \sim N_2(\beta^{(b)}, P)\) com as seguintes sugestões para \(P\):
    1. Em caso de fraca correlação a posteriori entre \(\beta_0\) e \(\beta_1\), pode tomar-se \(P=\text{diag}(p_0,p_1)\) em que \(\{p_j\}\) são estimativas preliminares das variâncias a posteriori de \(\{\beta_j\}\);
    1. No caso contrário, pode-se tomar-se \(P\) proporcional a \([I_\star(\widehat{\beta})]^{-1}\) em que \(I_\star(\beta)\) é o simétrico da matriz hessiana de \(\bar{h}(\beta|y)\), com a constante escolhida de modo a obter-se uma razoável taxa de aceitação.
  1. Rácio de M-H: \(R(u,v) = \frac{\bar{h}(v|y)}{\bar{h}(u|y)}\) (expressão a derivar).

9.5

  1. \(\pi_i(u_i|u_j) \sim \text{Exp}(\alpha+\beta\,u_j)\), \(i,j=1,2\), \(i\not=j\).

  2. Ter em conta que a função de transição de \(u=(u_1,u_2)\) para \(v=(v_1,v_2)\) em cada ciclo é \(p(u,v) = \pi_1(v_1|u_2) \pi_2(v_2|v_1)\).

9.6

Notar que as entradas \(q(u,v)\) de \(Q\) são tais que \[ q(u,v) = \begin{cases} 1/10, &u=(0,0) \ \text{e} \ v=(1,1), \\ 2/9, &u=(1,1) \ \text{e} \ v=(0,0). \end{cases} \]

9.7

  1. \(\beta_0|\beta_1,\sigma^2,y \sim N(\widehat{\beta}_0-\bar{x}(\beta_1-\widehat{\beta}_1), \sigma^2/n)\), \(\beta_1|\beta_0,\sigma^2,y \sim N(\widehat{\beta}_1 - \frac{n\,\bar{x}}{\sum_i x_i^2} (\beta_0 - \widehat{\beta}_0), \sigma^2/(\sum_i x_i^2))\), \(\sigma^2|\beta,y \sim GaI( \frac{n}{2}, \frac{(n-2)\,s^2 + (\beta-\widehat{\beta})' X'X (\beta-\widehat{\beta})}{2})\).

  2. Resposta negativa (reveja Cap. 4).

9.8

  1. \(\delta|\alpha,x \sim Ga(a+n,b+\sum_i x_i^\alpha)\), \(h(\alpha|\delta,x) \propto \alpha^{n-1} (\prod_i x_i)^\alpha \exp\{-\frac{1}{2\,d} (\ln\alpha-c)^2 - \delta \sum_i x_i^\alpha\}\).

9.9

  1. \(\eta|\delta,\mu,\tau,D \sim Ga(\sum_i y_i, \sum_i \delta^{x_i})\), \(D=\{(y_i,x_i)\}\); \(h(\delta|\eta,\mu,\tau,D) \propto \delta^{\sum_i x_i\,y_i-1} \exp(-\eta\,\sum_i \delta^{x_i})\); \(\mu|\eta,\delta,\tau,D \sim N(\bar{x}, (n\,\tau)^{-1})\); \(\tau|\eta,\delta,\mu,D \sim Ga(\frac{n}{2}, \frac{\sum_i (x_i-\bar{x})^2 + n(\mu-\bar{x})^2}{2})\).

9.10

\(\{\lambda_i\}|\beta,\{x_i\} \sim \prod_i Ga(a+x_i, t_i+\beta)\); \(h(\beta|\{\lambda_i\},\{x_i\}) \propto \beta^{n\,a-(c+1)} \exp(-\beta\sum_i \lambda_i - \frac{d}{\beta})\)
\(\therefore\ \) membro de uma distribuição Gaussiana Inversa Generalizada com parâmetros \(\nu=n\,a-c\), \(\chi=2\,d\), \(\psi=2\,\sum_i\lambda_i\) (analisar a sua log-concavidade).

9.11

  1. \(m_2|N,n_1,n_2,\theta \sim Hpg(N,n_1,n_2)\),
    \(h(N|n_1,n_2,m_2) \propto \frac{(N-n_1)! (N-n_2)!}{N! (N-n_1-n_2+m_2)!}\), \(N\geq n_1+n_2-m_2\).

9.12

\(\delta|\theta,N,D \sim Be(m_2+1, n_1-m_2+1)\), \(D=(n_1, m_2, n_c=n_1+n_2-m_2)\)
\(\theta|\delta,N,D \sim Be(n_c+1, 2\,N-n_1-n_c+1)\),
\(N|\delta,\theta,D\) tal que \(N-n_c|\delta,\theta,D \sim BiN(n_c, 1-(1-\theta)^2)\).

9.13

  1. \(\mu|\{y_{ij}\},\{\mu_i\},\sigma_{\epsilon}^2,\sigma_{\mu}^2 \sim N(c,\frac{\sigma_0^2\,\sigma_{\mu}^2}{\sigma_{\mu}^2 + k\,\sigma_0^2})\), \(c = \frac{\frac{k}{\sigma_{\mu}^2} \bar{\mu} + \frac{1}{\sigma_0^2} \mu}{\frac{k}{\sigma_{\mu}^2} + \frac{1}{\sigma_0^2}}\), \(\bar{\mu} = \frac{1}{k} \sum_i \mu_i\).

  2. \(\{\mu_i\}|\{y_{ij}\},\mu,\sigma_{\epsilon}^2,\sigma_{\mu}^2 \underset{ind.}{\sim} N(A_i,\frac{\sigma_{\epsilon}^2\,\sigma_{\mu}^2}{\sigma_{\epsilon}^2 + m\,\sigma_{\mu}^2})\), \(A_i = \frac{\frac{1}{\sigma_{\mu}^2} \mu + \frac{m}{\sigma_{\epsilon}^2} \bar{y}_{i\cdot}}{\frac{1}{\sigma_{\mu}^2} + \frac{m}{\sigma_{\epsilon}^2}}\).

  3. \(\sigma_{\epsilon}^2|\{y_{ij}\},\mu,\{\mu_i\},\sigma_{\mu}^2 \sim GaI(a_1+\frac{m\,k}{2}, b_1+\frac{1}{2} \sum_{i,j} (y_{ij}-\mu_i)^2)\).

  4. \(\sigma_{\mu}^2|\{y_{ij}\},\mu,\{\mu_i\},\sigma_{\epsilon}^2 \sim GaI(a_2+\frac{k}{2}, b_2+\frac{1}{2} \sum_{i} (\mu_{i}-\mu)^2)\).

9.14

\(\theta|\lambda,k,y \sim Ga(A_1,B_1)\), \(A_1=a_1+\sum_{i=1}^k y_i\), \(B_1=b_1+\sum_{i=1}^k t_i\).
\(\lambda|\theta,k,y \sim Ga(A_2,B_2)\), \(A_2=a_2+\sum_{i=k+1}^n y_i\), \(B_2=b_2+\sum_{i=k+1}^n t_i\).
\(h(k|\theta,\lambda,y) = \frac{p_k}{\sum_{j=1}^n p_j}\), \(k=1,\ldots,n\), \(p_k=(\frac{\theta}{\lambda})^{\sum_{j=1}^k y_j} \exp(-(\theta-\lambda) \sum_{j=1}^k t_j)\).

9.15

  1. \(z|\theta,x \sim Exp(b(\theta))\), \(b(\theta)=1+(\frac{\theta-\theta_0}{\sigma_0})^2\),
    \(\theta|z,x \sim N(\bar{\theta}, \sigma(z))\), \(\sigma(z) = (\frac{1}{\sigma^2} + \frac{2\,z}{\sigma_0^2})^{-1}\), \(\bar{\theta} = \frac{ \frac{1}{\sigma^2} x + \frac{2\,z}{\sigma_0^2} \theta_0}{\frac{1}{\sigma^2} + \frac{2\,z}{\sigma_0^2}}\).

9.16

\(z|\lambda,\delta,x \sim GaI(\frac{k+n}{2}, \frac{k\,\delta + n\,(\lambda-\bar{x})^2 + (n-1)\,s^2}{2})\),
\(\delta|\lambda,z,x \sim Ga(c+\frac{k}{2}, d+\frac{k}{2\,z})\),
\(\lambda|\delta,z,x \sim N(A(x,z), \frac{b^2\,z}{z+n\,b^2})\), \(A(x,z) = \frac{z\,a+n\,b^2\,\bar{x}}{z+n\,b^2}\).

9.17

\(\alpha|n,z \sim Be(a_p+z_{\cdot\cdot}, b_p+N-z_{\cdot\cdot})\),
\(\sigma_u|n,z \sim Be(s_u+z_u, r_u+z_{\cdot\cdot}-z_u)\),
\(\varepsilon_u|n,z \sim Be(e_u+N-N_u-z_{\cdot\cdot}+z_u, f_u+N_u-z_u)\), \(u=1,2\).

9.18

Notar que \[ \begin{array}{lll} \gamma_{jk}^{(l)} &= &P(T_1=j, T_2=k|R_V=l) \\ &= &P(T_1=j|R_V=l) \, P(T_2=k|R_V=l) + (-1)^{j+k} C_{12}^{(l)}, \end{array} \] onde \(C_{12}^{(l)}\) é a covariância entre \(T_1\) e \(T_2\) sob \(R_V=l\), denotada por \(C_{12}^{(1)}=\sigma_{12}\) e \(C_{12}^{(0)}=\varepsilon_{12}\) de modo que \[ \gamma_{jk}^{(1)} = \begin{cases} \sigma_1\,\sigma_2 + \sigma_{12}, &j=k=1, \\ \sigma_1 (1-\sigma_2) - \sigma_{12}, &j=1,\,k=0, \\ (1-\sigma_1) \sigma_2 - \sigma_{12}, &j=0,\,k=1, \\ (1-\sigma_1) (1-\sigma_2) + \sigma_{12}, &j=k=0, \\ \end{cases} \] e \(\gamma_{jk}^{(0)}\) dadas por expressão análogas com \(\varepsilon_u\) e \(\pm \varepsilon_{12}\) no papel de \(\sigma_u\) e \(\pm \sigma_{12}\).

As distribuições a priori condicionais Uniformes em apropriados intervalos limitados inferiormente por \(0\) para \(\varepsilon_u\) e \(\pm \varepsilon_{12}\) correspondem para os respetivos coeficientes de correlação \(\rho_{12}^{(l)}\) a distribuições Uniformes em intervalos limitados superiormente por funções das sensibilidades ou das especificidades consoante \(l=1\) ou \(l=0\). Por exemplo, tomando \(\rho_\sigma \equiv \rho_{12}^{(l)} = \frac{\sigma_{12}}{\sqrt{\sigma_1\,(1-\sigma_1) \, \sigma_2\,(1-\sigma_2)}} = \frac{\sigma_{12}}{\sqrt{m_1\,m_2}}\), a distribuição a priori condicional é \[ \rho_\sigma|\sigma_1,\sigma_2 \sim Unif([0,g_\sigma(\sigma_1,\sigma_2)]) \] com \(g_\sigma(\sigma_1,\sigma_2) = \sqrt{\frac{\min\{m_1,m_2\}}{\max\{m_1,m_2\}}}\), \(m_1=\sigma_1\,(1-\sigma_2)\), \(m_2=\sigma_2\,(1-\sigma_1)\). Argumento análogo para \(\rho_\varepsilon = \rho_{12}^{(0)}\) com \(\{\varepsilon_u\}\) no papel de \(\{\sigma_u\}\) conduz a \(\rho_\varepsilon|\varepsilon_1,\varepsilon_2 \sim Unif([0,g_\varepsilon(\varepsilon_1,\varepsilon_2)])\).

Tomando \(\phi = (\alpha,\sigma_1,\sigma_2,\varepsilon_1,\varepsilon_2,\rho_\sigma,\rho_\varepsilon)\), tem-se \[ h(\phi|n,z) \propto h(\phi) \, f(n|\phi) \, f(z|n,\phi) \] com \(n|\phi \sim M_3(N,\{\theta_{jk}(\phi)\})\), \(z|n,\phi \sim \prod_{j,k} Bi(n_{jk}, \nu_{jk})\), \(\nu_{jk} = \frac{\alpha\,\gamma_{jk}^{(1)}}{\theta_{jk}}\), \(\sigma_{12} = \rho_\sigma \sqrt{m_1\,m_2}\), ou seja, \[ \begin{array}{lll} h(\phi|n,z) &\propto &\alpha^{A_p-1} (1-\alpha)^{B_p-1} \times \prod_{j,k} (\gamma_{jk}^{(1)})^{z_{jk}} \times \prod_{j,k} (\gamma_{jk}^{(0)})^{n_{jk}-z_{jk}} \\ & &\times \prod_{u} \sigma_u^{s_u-1} (1-\sigma_u)^{r_u-1} \varepsilon_u^{e_u-1} (1-\varepsilon_u)^{f_u-1} \times [g_\sigma(\sigma_1,\sigma_2) \, g_\varepsilon(\varepsilon_1,\varepsilon_2)]^{-1} \end{array} \] para \(\alpha,\sigma_u,\varepsilon_u \in [0,1]\), \(\rho_\sigma \in [0,g_\sigma]\), \(\rho_\varepsilon \in [0,g_\varepsilon]\), em que \(A_p=a_p+z_{\cdot\cdot}\) e \(B_p=b_p+N-z_{\cdot\cdot}\).

9.19

\[ \begin{array}{lll} h(\theta,\alpha|y) &\propto &(\sigma_1^2)^{-\frac{IJ}{2}-1} \, (\sigma_2^2)^{-\frac{I}{2}-1} \times \exp\{-\frac{1}{2\sigma_1^2} \sum_{i,j} (y_{ij}-\bar{y}_{i\cdot})^2\} \\ & &\times \exp\{-\frac{1}{2} \sum_{i} ( \frac{J}{\sigma_1^2} [\alpha_i-(\bar{y}_{i\cdot}-\mu)]^2 + \frac{\alpha_i^2}{\sigma_2^2})\}. \end{array} \]

\[ \begin{array}{lll} h(\theta|y) &\propto &h(\theta) \int f(y|\theta,\alpha) \, h(\alpha|\sigma_2^2)\,d\alpha \\ &\propto &(\sigma_1^2)^{-\frac{I(J-1)}{2}-1} \, (\sigma_2^2)^{-1} \, (\sigma_1^2+J\,\sigma_2^2)^{-\frac{I}{2}} \\ & &\times \exp\{-\frac{1}{2\sigma_1^2} \sum_{i,j} (y_{ij}-\bar{y}_{i\cdot})^2\} \, \exp\{-\frac{J}{2(\sigma_1^2+J\,\sigma_2^2)} \sum_{i} (\bar{y}_{i\cdot}-\mu)^2\} . \end{array} \]

9.20

  1. Denotando \(\{e_j,\,j=1,2,3\}\) os vetores da base canónica de \(\mathbb{R}^3\) (e.g. \(e_1=(1,0,0)\)) e sendo \(P(Z_i=e_j|p)=p_j\), \(\forall\,j\), \[ P(Z_i=e_j|p,\beta,x_i) \propto p_j \beta_j x_i \exp(-\frac{\beta_j}{2} x_i^2) \equiv \bar{p}_j (x_i) \] com a constante de proporcionalidade dada por \([\sum_{j=1}^3 \bar{p}_j (x_i)]^{-1}\).
    \(p|x,z \sim D_2 (a+N)\), \(N=(n_j=\sum_{i=1}^n I_{\{e_j\}}(z_i),\,j=1,2,3)\),
    \(\beta_j|x,z,j=1,2,3 \underset{ind.}{\sim} Ga(c_j+n_j, d_j+\frac{s_j^2}{2})\), \(s_j^2=\sum_{i=1}^n x_i^2 I_{\{e_j\}}(z_i)\).

9.21

  1. Notar que \(P(Y_L \in C\cap A) = P(Y_1 \in C \cap A)\) e \(P(Y_L \in C) = \frac{P(Y_1 \in C \cap A)}{P(Y_1 \in A)}\).

  2. Mostre que \(P(Y_L \in C) = \int_{C\cap A} \frac{1}{m(A)} d\,y_1\).

  3. \(B=[0,1]\times [0,M]\), \(M=\max_u \pi^\star(u)\), \(V \sim Unif([0,1])\) e \(W \sim Unif([0,M])\) independentes.

9.22

\(f(y|\{\delta_i\}) \propto a(\beta) \prod_{j=0,1} \text{e}^{m_j\,\beta_j}\), \(a(\beta) = \text{e}^{-\sum_i \phi_i(\beta)}\),
\(\beta \sim N_2\biggl(\begin{pmatrix} a_0 \\ a_1 \end{pmatrix}, \text{diag}(b_0^2,b_1^2)\biggr)\).

  1. \(h(\beta|y) \propto a(\beta) \prod_{j=0,1} g_j(\beta_j)\), \(g_j(\beta_j) = \exp(-\frac{1}{2} [\frac{1}{b_j^2} (\beta_j-a_j)^2 - 2\,m_j\,\beta_j])\).
    Tomar \(Z\) tal que \(Z|\beta,y \sim Unif(]0,a(\beta)[)\) \(\Leftrightarrow W = -\ln Z|\beta,y \sim Exp(1)\) deslocada de \(\sum_i \phi_i(\beta) \equiv \phi_\cdot(\beta)\)
    \[ \begin{array}{lll} f(w|\beta,y) &\propto &\exp(-w) \, I_{(\phi_\cdot(\beta),\infty)}(w), \\ h(\beta|w,y) &\propto &\prod_j g_j(\beta_j) \, I_{(\phi_\cdot(\beta),\infty)}(w) \propto \prod_j h_\star(\beta_j) \, I_{(\phi_\cdot(\beta),\infty)}(w) \end{array} \] em que \(h_\star(\beta_j) \sim N(a_j+m_j\,b_j^2, b_j^2)\)
    \(\therefore\) a distribuição condicional completa de cada \(\beta_j\) é uma Normal truncada com restrição a \(w>\phi_\cdot(\beta) \Leftrightarrow \sum_i E_i \text{e}^{\beta_1\,x_i} < \text{e}^{-\beta_0} w\).

9.23

  1. \(f(y|\beta) = \prod_{i=1}^n f(y_i|\beta) = \prod_i [\exp(\text{e}^{\delta_i})-1]^{y_i} \exp(-\text{e}^{\delta_i})\), \(\delta_i=\beta_0+\beta_1\,x_i\),
    \(Z=(Z_i,\,i=1,\ldots,n)\), \(\{Z_i\}|y,\beta \underset{ind.}{\sim} Unif(A_i)\), \(A_i=[0,f(y_i|\beta)]\)
    \(\Rightarrow \ f(z|y,\beta) = [\prod_{i=1}^n f(y_i|\beta)]^{-1} \prod_i I_{A_i}(z_i)\)
    \(\Rightarrow \ h(\beta,z|y) = \prod_j h(\beta_j) \prod_i I_{A_i}(z_i)\)
    \(\beta_j|\beta_{-j},z,y \sim N(a_j, b_j^2)\) restrita a \(S_j\) definida como se segue.
  • Caso \(y_i=1\): \(S(\beta) = \{\beta: 1-\exp(-\text{e}^{\beta_0+\beta_1\,x_i}) \geq z_i\}\)
    \(\Rightarrow\) Dado \(\beta_1\) e \(z_i\), \(\beta_0 \geq \ln [-\ln(1-z_i)] -\beta_1\,x_i \equiv L_{0i}\), \(\forall\,i\),
    \[ S_0: \beta_0 \geq \max_{i:y_i=1} L_{0i} \equiv L_0. \] \(\Rightarrow\) Dado \(\beta_0\) e \(z_i\), \(\beta_1\,x_i \geq \ln [-\ln(1-z_i)] -\beta_0\),
    1. \(\Leftrightarrow\) Se \(x_i>0\), \(\beta_1 \geq x_i^{-1} \{\ln [-\ln(1-z_i)] -\beta_0 \} \equiv L_{1i}^+\) \(\Rightarrow \ \beta_1 \geq \max_{i:y_i=1,x_i>0} L_{1i}^+ \equiv L_1^+\),
    1. \(\Leftrightarrow\) Se \(x_i<0\), \(\beta_1 \leq x_i^{-1} \{\ln [-\ln(1-z_i)] -\beta_0 \} \equiv U_{1i}^-\) \(\Rightarrow \ \beta_1 \geq \min_{i:y_i=1,x_i<0} U_{1i}^- \equiv U_1^-\).
      \({}^{}\)
  • Caso \(y_i=0\): \(S(\beta) = \{\beta: \exp(-\text{e}^{\beta_0+\beta_1\,x_i}) \geq z_i\} = \{\beta: \beta_0+\beta_1\,x_i \leq \ln[-\ln z_i]\}\)
    \(\Rightarrow\) Dado \(\beta_1\) e \(z_i\), \(\beta_0 \leq \ln [-\ln z_i] -\beta_1\,x_i \equiv U_{0i}\), \(\forall\,i\), \[ S_0: \beta_0 \leq \min_{i:y_i=0} U_{0i} \equiv U_0. \] \(\Rightarrow\) Dado \(\beta_0\) e \(z_i\), \(\beta_1\,x_i \leq \ln [-\ln z_i] -\beta_0\),
    1. \(\Leftrightarrow\) Se \(x_i>0\), \(\beta_1 \leq x_i^{-1} \{\ln [-\ln z_i] -\beta_0 \} \equiv U_{1i}^+\) \(\Rightarrow \ \beta_1 \leq \min_{i:y_i=0,x_i>0} U_{1i}^+ \equiv U_1^+\)
    1. \(\Leftrightarrow\) Se \(x_i<0\), \(\beta_1 \geq x_i^{-1} \{\ln [-\ln z_i] -\beta_0 \} \equiv L_{1i}^-\) \(\Rightarrow \ \beta_1 \geq \max_{i:y_i=0,x_i<0} L_{1i}^- \equiv L_1^-\).

Em suma:

  1. \(\beta_0|\beta_1,z,y \sim N(a_0,b_0^2)\) restrita a \(L_0 \leq \beta_0 \leq U_0\);
  2. Em \(\{i:x_i>0\}\), \(L_1^+ \leq \beta_i \leq U_1^+\) e em \(\{i:x_i<0\}\), \(L_1^- \leq \beta_i \leq U_1^-\) \(\Rightarrow \ L_1 \equiv \max\{L_1^+,L_1^-\} \leq \beta_1 \leq \min\{U_1^+,U_1^-\} \equiv U_1\),
    \(\beta_1|\beta_0,z,y \sim N(a_1,b_1^2)\) restrita a \(L_1 \leq \beta_1 \leq U_1\).
  1. Seguir o argumento usado em a).

Capítulo 10

10.2

Ter em consideração as sugestões explicitadas no enunciado.

10.3

  1. Prove primeiro que \(Cov(\alpha_i,\bar{\alpha}|y) = C + \frac{W}{I} = Var(\bar{\alpha}|y)\),
    \(Var(\alpha_i^\star|y) = \frac{I-1}{I} W\),
    \(Cov(\alpha_i^\star,\alpha_j^\star|y) = -\frac{W}{I}\), \(i\not= j\) em que \(C=Cov(\alpha_i,\alpha_j|y)\) e \(W\) é o valor comum dos \(W_i\).

  2. \(\rho(\beta_i,\beta_j|y) = 1/2\), \(i,j\geq 2\), \(i\not=j\)
    \(\rho(\beta_i,\nu|y) = - [2\,(1+\frac{\sigma_1^2}{IJ\sigma_2^2})]^{-\frac{1}{2}}\), como consequência de \(Cov(\beta_i,\nu|y) = - W\) e \(Var(\nu|y) = \frac{W}{\sigma_2^2} (\sigma_2^2 + \frac{\sigma_1^2}{IJ})\).

10.4

Pela Proposição 10.3 as duas distribuições condicionais não são compatíveis pelo que não existe uma f.d.p. conjunta própria com elas associada. A aplicação do algoritmo Gibbs com aquelas distribuições mostrará que a cadeia nunca atingirá a convergência.

10.7

Tendo em conta que \(\beta|v \sim N_p(0,B_v)\), com \(B_v=\text{diag}(I_jb_j^2, j=1,\ldots,p)\), \(I_j=(1-v_j)+v_j\,a_j^2\), e a identidade algébrica da combinação de duas formas quadráticas no caso multivariado (Cap.4): \[ \beta|\sigma^2,v,y \sim N_p(c,C),\ c = (\frac{X'X}{\sigma^2} + B_v^{-1})^{-1} \frac{X'X}{\sigma^2} \widehat{\beta}, \]
com \(\widehat{\beta}\) a estimativa de MV de \(\beta\) e \(C = \frac{X'X}{\sigma^2} (\frac{X'X}{\sigma^2} + B_v^{-1})^{-1} B_v^{-1} \equiv [\sigma^2 (X'X)^{-1} + B_v]^{-1}\) \[ \sigma^2|\beta,v,y \sim GaI(d_v+\frac{n}{2}, f_v + (y-X\beta)'(y-X\beta)). \] No caso em que \(d_v\) e \(f_v\) não dependem de \(v=\{v_j\}\), \(h(v|\beta,\sigma^2,y) = \prod_{j=1}^p Ber(\frac{p_j}{p_j+q_j})\) com \(p_j=w_j \exp(-\frac{\beta_j^2}{2\,a_j^2b_j^2})\), \(q_j=(1-w_j) \exp(-\frac{\beta_j^2}{2\,b_j^2})\).

10.8

\(\alpha(j,j') = \min\{1,R(j,j')\}\) com \[ R(j,j') = \frac{h(M_{j'}) f(y|M_{j'}) q_m(M_j|M_{j'})}{h(M_{j}) f(y|M_{j}) q_m(M_{j'}|M_{j})}, \] atendendo a que a distribuição proponente \(q(\cdot)\) para \((M_j,\theta_j) \rightarrow (M_{j'},\theta_{j'})\) é o produto dos fatores \(q_m(M_j|M_{j'})\) independente de \(\theta\), e \(q_p(\theta_{j'}|M_{j'},M_j,\theta_j) = h(\theta_{j'}|M_{j'},y)\).
Como o rácio M-H apresenta a estrutura de um amostrador de dimensão fixa sobre o espaço \({\cal M}\) dos modelos, o correspondente algoritmo segue a versão padrão.

10.9

\(\theta_1=(\alpha,\lambda)\), \(\theta_2=(\mu,\sigma^2)\),
\(\mu_1(M_1) \equiv E(Y|\theta_1)=\alpha/\lambda\),
\(\mu_2(M_1) \equiv E(Y^2|\theta_1)=\frac{\alpha(\alpha+1)}{\lambda^2}\),
\(\mu_2(M_2) \equiv E(Y|\theta_2)=\text{e}^{\mu+\sigma^2/2}\),
\(\mu_2(M_2) \equiv E(Y^2|\theta_2)=\text{e}^{2(\mu+\sigma^2)}\),
\(\mu_k(M_1)=\mu_k(M_2)\), \(k=1,2\)
\(\Leftrightarrow \ \theta_2 = (\ln \frac{\alpha/\lambda}{(1+\frac{1}{\alpha})^{1/2}}{}, \ln (1+\frac{1}{\alpha})) \equiv (\mu^\star(\theta_1), \sigma_\star^2(\theta_1)) \equiv \theta_2^\star(\theta_1)\)
\(\Leftrightarrow \ \theta_1 = ([\text{e}^{\sigma^2}-1]^{-1}, [\text{e}^{\mu+\sigma^2/2} (\text{e}^{\sigma^2}-1)]^{-1}) \equiv \theta_1^\star(\theta_2)\).
Com as perturbações estocásticas as relações ficam
\(\theta_2 = (\mu^\star(\theta_1) + w_1, \sigma^2_\star (\theta_1) \times w_2) \equiv g_{12}^p(\theta_1,w)\) em que \(w=(w_1,w_2) \sim q_p(\cdot|M_1,\theta_1)\) \[ \begin{array}{lll} \theta_1 &= &(\alpha^\star(\mu-w_1', \sigma_2/w_2'), \lambda^\star(\mu-w_1', \sigma_2/w_2')) \\ &= & ([\text{e}^{\sigma^2/w_2'}-1]^{-1}, [\text{e}^{\mu-w_1'+\sigma^2/(2\,w_2')} (\text{e}^{\sigma^2/w_2'}-1)]^{-1}) \equiv g_{21}^p(\theta_2,w') \end{array} \] com \(w'=(w_1',w_2') \sim q_p(\cdot|M_2,\theta_2)\).
Considerando \(w'=g_{12}^e(\theta_1,w)\), obtém-se uma transformação biunívoca entre \((\theta_1,w)\) e \((\theta_2,w')\) expressa por \((\theta_2,w')=g_{12}(\theta_1,w)\) com a correspondente função diferenciável tomada como \(g_{12}(\cdot)=(g_{12}^p(\cdot), g_{12}^e(\cdot))\).
O jacobiano da transformação \((\theta_2,w')=g_{12}(\theta_1,w)\) é pela fórmula de Laplace \(|\frac{\partial g_{12}(\theta_1,w)}{\partial (\theta_1,w)}| = w_2 [\lambda \alpha (\alpha+1)]^{-1}\).

10.11

Sendo \(\alpha=(\theta,\lambda,k,b_1,b_2)\) o vetor de parâmetros do modelo bayesiano e usando a notação \(\theta^-\), por exemplo, para indicar os parâmetros de \(\alpha\) à exceção de \(\theta\),as distribuições condicionais completas são,
\(\theta|y,\theta^- \sim Ga(A_1,B_1)\), \(A_1=\sum_{i=1}^k y_i + a_1\), \(B_1=\sum_{i=1}^k t_i + b_1\),
\(\lambda|y,\lambda^- \sim Ga(A_2,B_2)\), \(A_2=\sum_{i=k+1}^n y_i + a_2\), \(B_2=\sum_{i=k+1}^n t_i + b_2\),
\(h(k|y,k^-) = \frac{\bar{p}_k}{\sum_{j=1}^n \bar{p}_j}\), \(\bar{p}_j = (\frac{\theta}{\lambda})^{\sum_{i=1}^j y_i} \text{e}^{-(\theta-\lambda) \sum_{i=1}^j t_i}\), \(j=1,\ldots,n\),
\(b_1|y,b_1^- \sim Ga(a_1+c_1,d_1+\theta)\),
\(b_2|y,b_2^- \sim Ga(a_2+c_2,d_2+\lambda)\).

10.13

Tomar em consideração que para o modelo Weibull geral com \(\beta_4>0\), \(\lim_{x\to \infty} \mu(x) = \beta_1\), \(\lim_{x\to 0} \mu(x) = \beta_1-\beta_2\) e que \(\ln[-\ln \frac{\beta_1-\mu(x)}{\beta_2}] = \ln \beta_3 + \beta_4\ln x\) e aplicar o mesmo tipo de argumento mutatis mutandis aos modelos em comparação.
A verificação das diferenças entre as duas distribuições a priori para cada modelo deve passar pela análise de medidas de desempenho preditivo e de diagnóstico, por um lado, e de estimativas paramétricas por outro.